Past Paper - 221208 - 003047
Past Paper - 221208 - 003047
Past Paper - 221208 - 003047
1st year:
1. Which of the following does not pass through the inguinal canal?
A. Round ligament
B. Vas deferens
C. Genital branch of genitofemoral nerve
D. Broad ligament
E. Testicular artery
ANSWER: D
2. Four days after an Ivor Lewis esophagectomy, you start enteral feeding through the jejunostomy feeding
tube. The long chain fatty acids contained in the feeds:
A. Are not found in chylomicrons
B. Are only synthesized in the body
C. Enter the circulation via lymphatics
D. Enter the circulation via the portal system
E. Are not absorbed at all
ANSWER: C
3. A 65-year-old woman, presents with a tender mass below and medial to the pubic tubercle. One Of the
following is not a boundary to this mass:
A. Femoral artery
B. Femoral vein
C. Inguinal ligament
D. Pectineal ligament
E. Lacunar ligament
ANSWER: A
5. A patient who previously sustained a large open gluteal laceration, comes to your clinic with a smaller
wound to your surprise. This is explained by one of the following:
A. Lymphocytes
B. Macrophages
C. Fibroblasts
D. Myofibroblasts
E. PMNs
ANSWER: D
10. Which of the following body fluids contains the highest concentration of bicarbonate?
A. Stomach.
B. Small intestine.
C. Colon.
D. Pancreas.
E. Bile.
ANSWER: D
12. Which of the following has the highest effect in increasing nitrogen wasting?
A. Major burn.
B. Skeletal trauma.
C. Severe sepsis.
D. Elective surgery.
E. Starvation.
ANSWER: A
15. In which region of the GI tract is the longitudinal muscle of the muscularis arranged into distinct longitudinal
bundles?
A. Duodenum
B. Jejunum
C. Ileum
D. Colon
E. stomach
ANSWER: D
17. Which of the following is contraindication for day-case inguinal hernia surgery?
A. Age of 70 years old.
B. Well controlled epilepsy.
C. Well control Diabetes type 1.
D. American Society Anesthesiologist (ASA) score 3.
E. Body mass index of 35kg/m2.
ANSWER D
19. In response to reversible hypovolemic shock, all of the following indices decrease except
A. Cardiac output.
B. Systemic vascular resistance.
C. Blood volume in venous capacitance.
D. Central venous pressure.
E. Mixed venous oxygen saturation.
ANSWER: B
20. In an average adult, all the following are compatible with class II hemorrhage except
A. Blood loss of 1000cc.
B. Heart rate 130 per minute.
C. Postural hypotension.
D. Anxiety.
E. Decreased central venous pressure.
ANSWER: B
21. A24-year-old woman is scheduled for an elective cholecystectomy. The best method of identifying a
potential bleeding is which of the following?
A. Platelet count
B. A complete history and physical examination
C. Bleeding time
D. Lee-White clotting time
E. Prothrombin time (PT)
ANSWER: B
22. After undergoing subtotal gastrectomy for carcinoma of the stomach, a 64-year-old woman is receiving
peripheral parenteral nutrition. To increase calories by the peripheral route, what should be prescribed?
A. D5W in normal saline
B. Multivitamin infusion
C. D25W (25% dextrose in water)
D. Soybean oil (intra lipid)
E. Lactulose
ANSWER: D
24. In response to trauma, the serum levels of all of the following hormones increase except
A. T3.
B. Adrenalin.
C. Insulin.
D. Growth hormone.
E. Cortisol.
ANSWER: C
25. All of the following increase the amount of insensible water loss except.
A. Dry weather.
B. Fever.
C. Diarrhea.
D. Tracheotomy.
E. Hot weather.
ANSWER: C
28. Which of the following is an advantage of transjugular central venous line over a central venous line
inserted through a peripheral vein?
A. We can give higher concentration of glucose solutions.
B. We can keep it for a longer period.
C. Less risk of pneumothorax.
D. Less risk of arrythmia.
E. Less risk of air embolism.
ANSWER: B
29. In the management of a patient who has massive trauma and hypotension, all of the following should be
done except
A. Stop hemorrhage.
B. Correct coagulopathy.
C. Elevate systolic pressure above 110mmHg.
D. Control sepsis or major contamination.
E. Give blood.
ANSWER: C
31. All of the following are steps done in processing a specimen for histological examination except
A. Fixation in formalin for one day.
B. Make representative tissue blocks.
C. Place blocks in paraffin wax.
D. Make thin slices on glass slides.
E. Stain slides with Indian ink.
ANSWER: E
32. All of the following are of the first step of pain stepladder booklet advised by WHO to control chronic pain
caused by malignancy, except
A. Tramadol.
B. Aspirin.
C. Paracetamol.
D. Nonsteroidal anti-inflammatory drugs.
E. Tricyclic antidepressants.
ANSWER: A
33. Measures to reduce the risk of venous thromboembolism include all of the following except
A. Prevent preoperative dehydration.
B. Start to apply graduated compressing lower limbs stockings immediately after surgery.
C. Apply intraoperative foot impulse device.
D. Use intraoperative pneumatic compressing device.
E. Start prophylactic low molecular weight heparin preoperatively.
ANSWER: B
34. All of the following combinations between an ulcer’s edge and pathology are true except
A. Sloping edge: healing ulcer.
B. Undermined edge: bed sore.
C. Rolled edge: basal cell carcinoma.
D. Everted edge: tuberculosis.
E. Punched-out edge: syphilis.
ANSWER: D
35. All of the following are proven examples of congenital sinuses except
A. Preauricular.
B. Umbilical.
C. Pilonidal.
D. Urachus.
E. Coccygeal.
ANSWER: C
37. All of the following should be checked before induction of anesthesia in a patient planed for paraumbilical
hernia repair except
A. Confirm the patient’s name.
B. Confirm the diagnosis.
C. Check the consent.
D. Mark the site of the procedure.
E. Ask for known allergies.
ANSWER: D
39. Which of the following organs creates the least immunological response in deceased donor organ
transplantation?
A. Liver.
B. Kidney.
C. Cornea.
D. Pancreas.
E. Small bowel.
ANSWER: C
40. All the following are true about trichilemmal cyst except
A. More common in the scalp.
B. Has a distinct odor if ruptured.
C. Has a thin wall.
D. The wall is a complete mature epidermis with granular layer.
E. If causes symptoms, the entire cyst is removed.
ANSWER: D
43. All the following are indications for insertion of a Folly’s catheter except
A. A patient with sever pancreatitis.
B. A female trauma patient who has femur fracture.
C. A male trauma patient who has blunt liver injury.
D. A child admitted for acute appendicitis.
E. A male presented to the emergency room with urine retention.
ANSWER: D
44. A 34-year-old female has left thyroid nodule about 2 by 3 cm. Which is the best pre-operative diagnostic
modality?
A. Fine needle aspiration cytology.
B. True-cut biopsy.
C. Incisional biopsy.
D. TSH serum level.
E. Thyroid isotope scan.
ANSWER: A
45. A 25-year-old female underwent total thyroidectomy for Graves disease, 8 hours after surgery she informed
the nurses that she has numbness in her face and fingers. This complaint is most likely caused by
A. Hypothyroidism.
B. Hypocalcemia.
C. Recurrent laryngeal nerve injury.
D. Neck hematoma.
E. Anesthesia induced neuropathy.
ANSWER: B
46. Which of the following is a cause of wide anion gape metabolic acidosis?
A. Nasogastric tube.
B. Diabetic ketoacidosis.
C. Pancreatic fistula.
D. Diarrhea.
E. Chronic renal failure.
ANSWER: B
47. Which of the following surgeries carries the highest risk for deep vein thrombosis?
A. Surgery for subdural hemorrhage.
B. Liver resection for tumor.
C. Hysterectomy.
D. Total hip replacement.
E. Nephrectomy.
ANSWER: D
48. A 20-year-old male presented with an asymptomatic, 4 by 4 cm soft mass in his abdominal wall, not
attached to skin. The most probable diagnosis is
A. Lipoma.
B. Sebaceous cyst.
C. Soft tissue sarcoma.
D. Accessory breast.
E. Intradermal nevus.
ANSWER: A
49. The surgeon did total gastrectomy for gastric cancer, before closing the abdomen he noticed significant
change in the color of the left lateral segment of the liver. This mostly indicates the presence of
A. Replaced right hepatic artery.
B. Accessory right hepatic artery.
C. Replaced left hepatic artery.
D. Accessory left hepatic artery.
E. Replaced common hepatic artery.
ANSWER: C
50. All the following features are characteristic of dysplastic nevus except
A. Variegated pigmentation.
B. Ill-defined borders.
C. Irregular surface.
D. Size more than 5 mm in diameter.
E. Blue color.
ANSWER: E
51. Which of the following is contraindication for day-case inguinal hernia surgery?
A. Age of 70 years old.
B. Well controlled epilepsy.
C. Well control Diabetes type 1.
D. American Society Anesthesiologist (ASA) score 3.
E. Body mass index of 35kg/m2.
ANSWER D
53. In response to reversible hypovolemic shock, all of the following indices decrease except
A. Cardiac output.
B. Systemic vascular resistance.
C. Blood volume in venous capacitance.
D. Central venous pressure.
E. Mixed venous oxygen saturation.
ANSWER: B
54. In an average adult, all the following are compatible with class II hemorrhage except
A. Blood loss of 1000cc.
B. Heart rate 130 per minute.
C. Postural hypotension.
D. Anxiety.
E. Decreased central venous pressure.
ANSWER: B
56. A 20 years old male patient, presented complaining of abdominal pain and vomiting for 24 hours due to
intestinal obstruction, he is thirsty with dry mouth, mucous membranes and axillae, he has mild tachycardia
but normal supine blood pressure, his weight is 80kg. Which of the following parenteral fluid therapy
formulas may be more suitable for him in the first 24 hours?
A. 8 L glucose saline 0.45%.
B. 1 L normal saline and 2 L glucose water 5%.
C. 2 L normal saline and 1 L glucose water 5%.
D. 4 L glucose saline 0.18%.
E. 4 L normal saline.
ANSWER: A
57. All the following are true about liver anatomy except
A. Cantlie’s line extends roughly between the gallbladder fossa and the inferior vena cava.
B. The right lobe accounts around two thirds of the liver size.
C. The Caudate lobe lies posterior to the level of the porta hepatis.
D. The anterior branch of the right portal vein usually supplies liver segments V and VI.
E. The right hepatic vein drains venous tributaries from segments V, VI, VII and VIII.
ANSWER: D
58. All the following are true regarding the pancreatic ducts except
A. The duct of Santorini embryologically comes from the dorsal pancreatic bud.
B. The ventral bud forms most of the pancreas.
C. In most people most of the pancreas is drained via the major ampulla.
D. In most people the two pancreatic ducts are communicated.
E. The ampulla of Vator located between the proximal two thirds and the distal one third of the second part of
duodenum.
Answer: B
59. Which of the following does not pass through the diaphragmatic openings?
A. Aorta
B. Inferior Vena Cava
C. Esophagus
D. Left gastric artery
E. Left phrenic nerve
ANSWER: D
60. 10. In a total thyroidectomy for advanced papillary cancer, you notice the tumor on the right is adherent to a
nerve in the tracheo-esophageal groove. This nerve injury or resection will result in:
A. Loss of high-pitched voice
B. Stridor with need for tracheostomy
C. Unilateral right vocal cord paralysis
D. Loss of sensation above the vocal cords.
E. Loss of sensation in the skin of the right muscular triangle
ANSWER: C
61. A patient asks you when he can allow water to touch his closed surgical wound. Which of the following is
true pertaining to wound healing& external environment?
A. One week, as epithelialization and granulation tissue formation need to occur to protect the wound further,
even with meticulous closure
B. 24 hours, as meticulous closure with epithelialization seals the wound from external environment, in the
absence of excessive tension on wound
C. Two weeks, as acellular collagen starts forming and maturation phase occurs
D. Five days, as epithelialization requires this long, and collagen formation begins then
E. Three weeks to allow collagen type III, which is present in normal skin, to fully form
ANSWER: B
63. Which of the sutures listed below would be most suitable for suturing the femoral artery following an
embolectomy?
A. 5/0 polyglactin
B. 5/0 polypropylene
C. 5/0 silk
D. 5/0 polyglycolic acid
E. 5/0 polyester
ANSWER: B
64. A24-year-old woman is scheduled for an elective cholecystectomy. The best method of identifying a
potential bleeding is which of the following?
A. Platelet count
B. A complete history and physical examination
C. Bleeding time
D. Lee-White clotting time
E. Prothrombin time (PT)
ANSWER: B
65. A 60-year-old woman who underwent a mastectomy for breast cancer 2 years earlier presents to the
emergency department with headache, backache, and frequent vomiting. She is extremely thirsty and
stuporous. Which test is most likely to identify the cause?
A. CT scan of the head
B. X-ray of spine
C. Serum sodium determination
D. Serum calcium determination
E. Serum glucose determination
ANSWER: D
66. A 43-year-old woman with von Willebrand’s disease is scheduled for cholecystectomy. It can be stated that
preoperative evaluation will reveal which of the following?
A. Normal bleeding time, PT, and PTT
B. Platelet aggregate with restocetin
C. Increased bleeding time and PTT, and normal PT
D. Increased bleeding time and PT, and normal PTT
E. Increased bleeding time, and normal PT and PTT
ANSWER: C
67. A 64-year-old woman undergoing radical gastrectomy under general anesthesia is transfused with 2 U of
packed RBCs. A hemolytic transfusion reaction during anesthesia will be characterized by which of the
following?
A. Shaking chills and muscle spasms
B. Fever and oliguria
C. Hyperpyrexia and hypotension
D. Tachycardia and cyanosis
E. Bleeding and hypotension
ANSWER: E
68. After undergoing subtotal gastrectomy for carcinoma of the stomach, a 64-year-old woman is receiving
peripheral parenteral nutrition. To increase calories by the peripheral route, what should be prescribed?
A. D5W in normal saline
B. Multivitamin infusion
C. D25W (25% dextrose in water)
D. Soybean oil (intra lipid)
E. Lactulose
ANSWER: D
72. All of the following are true regarding human body fluids except
A. Water constitutes about 60% of the weight of an average adult.
B. Most of body water presents intracellular.
C. An average adult, about 75 kg weight, needs about 1400cc of water daily.
D. Urine constitutes the major source of water loss.
E. Interstitial water is much more than intravascular water.
ANSWER: C
73. In response to trauma, the serum levels of all of the following hormones increase except
A. T3.
B. Adrenalin.
C. Insulin.
D. Growth hormone.
E. Cortisol.
ANSWER: C
74. All of the following increase the amount of insensible water loss except.
A. Dry weather.
B. Fever.
C. Diarrhea.
D. Tracheotomy.
E. Hot weather.
ANSWER: C
77. All the following are true regarding respiratory alkalosis except
A. Caused by increase respiratory CO2 washing.
B. Kidneys compensate rapidly for the condition.
C. Compensation never overcorrect the pH.
D. Rebreathing mask can be used in treatment.
E. May be associated by carpopedal spasm.
ANSWER: B
80. Which of the following blood products carries the highest risk of transmission of bacterial infection?
A. Packed RBCs.
B. Fresh frozen plasma.
C. Platelets concentrates.
D. Cryoprecipitate.
E. Whole blood.
ANSWER: C
81. Which of the following is an advantage of transjugular central venous line over a central venous line
inserted through a peripheral vein?
A. We can give higher concentration of glucose solutions.
B. We can keep it for a longer period.
C. Less risk of pneumothorax.
D. Less risk of arrythmia.
E. Less risk of air embolism.
ANSWER: B
82. In the management of a patient who has massive trauma and hypotension, all of the following should be
done except
A. Stop hemorrhage.
B. Correct coagulopathy.
C. Elevate systolic pressure above 110mmHg.
D. Control sepsis or major contamination.
E. Give blood.
ANSWER: C
83. In comparison between healing by primary and secondary intentions for a wound involving skin and
subcutaneous fat, healing by primary intension has
A. More inflammatory response.
B. Less tensile strength.
C. Less chance to develop abscess.
D. Less scar.
E. More granulation tissue formation.
ANSWER: D
84. All of the following are true about polypropylene sutures except
A. It is monofilament.
B. It preserves its tensile strength for many years.
C. It is non absorbable.
D. It develops high tissue reaction.
E. It has high memory.
ANSWER: D
85. All of the following increase the risk of wound infection except
A. Obesity.
B. Jaundice.
C. Uremia.
D. Chemotherapy.
E. Pathing at the first postoperative day.
ANSWER: E
86. Which of the following sutures carries a higher risk of wound infection?
A. Silk.
B. Polypropylene.
C. Nylon.
D. Polyglactin.
E. Polydioxanone.
ANSWER: A
89. All of the following are of the first step of pain stepladder booklet advised by WHO to control chronic pain
caused by malignancy, except
A. Tramadol.
B. Aspirin.
C. Paracetamol.
D. Nonsteroidal anti-inflammatory drugs.
E. Tricyclic antidepressants.
ANSWER: A
90. Measures to reduce the risk of venous thromboembolism include all of the following except
A. Prevent preoperative dehydration.
B. Start to apply graduated compressing lower limbs stockings immediately after surgery.
C. Apply intraoperative foot impulse device.
D. Use intraoperative pneumatic compressing device.
E. Start prophylactic low molecular weight heparin preoperatively.
ANSWER: B
91. All of the following combinations between an ulcer’s edge and pathology are true except
A. Sloping edge: healing ulcer.
B. Undermined edge: bed sore.
C. Rolled edge: basal cell carcinoma.
D. Everted edge: tuberculosis.
E. Punched-out edge: syphilis.
ANSWER: D
92. All of the following are proven examples of congenital sinuses except
A. Preauricular.
B. Umbilical.
C. Pilonidal.
D. Urachus.
E. Coccygeal.
ANSWER: C
94. All of the following should be checked before induction of anesthesia in a patient planed for paraumbilical
hernia repair except
A. Confirm the patient’s name.
B. Confirm the diagnosis.
C. Check the consent.
D. Mark the site of the procedure.
E. Ask for known allergies.
ASWER: D
97. All the following are true about venous ulcers except
A. Associated with varicose veins.
B. Usually on the lateral aspect of the leg.
C. Usually shallow.
D. The affected limb is usually warm.
E. More common in women.
ANSWER: B
98. Which of the following organs creates the least immunological response in deceased donor organ
transplantation?
A. Liver.
B. Kidney.
C. Cornea.
D. Pancreas.
E. Small bowel.
ANSWER: C
99. All the following are true about trichilemmal cyst except
A. More common in the scalp.
B. Has a distinct odor if ruptured.
C. Has a thin wall.
D. The wall is a complete mature epidermis with granular layer.
E. If causes symptoms, the entire cyst is removed.
ANSWER: D
101. All the following are true about laryngeal mask airway except
A. It sits in the oropharynx.
B. It is passed to its site under vision.
C. It has a cuff which is inflated to push the soft tissue away from the laryngeal inlet.
D. It does not pass through the vocal cords.
E. It does not fully protect against aspiration.
ANSWER: B
102. Which of the following is contraindication for day-case inguinal hernia surgery?
A. Age of 70 years old.
B. Well controlled epilepsy.
C. Well control Diabetes type 1.
D. American Society Anesthesiologist (ASA) score 3.
E. Body mass index of 35kg/m2.
ANSWER D
104. In response to reversible hypovolemic shock, all of the following indices decrease except
A. Cardiac output.
B. Systemic vascular resistance.
C. Blood volume in venous capacitance.
D. Central venous pressure.
E. Mixed venous oxygen saturation.
ANSWER: B
105. In an average adult, all the following are compatible with class II hemorrhage except
A. Blood loss of 1000cc.
B. Heart rate 130 per minute.
C. Postural hypotension.
D. Anxiety.
E. Decreased central venous pressure.
ANSWER: B
107. A 20 years old male patient, presented complaining of abdominal pain and vomiting for 24 hours due to
intestinal obstruction, he is thirsty with dry mouth, mucous membranes and axillae, he has mild tachycardia
but normal supine blood pressure, his weight is 80kg. Which of the following parenteral fluid therapy
formulas may be more suitable for him in the first 24 hours?
A. 8 L glucose saline 0.45%.
B. 1 L normal saline and 2 L glucose water 5%.
C. 2 L normal saline and 1 L glucose water 5%.
D. 4 L glucose saline 0.18%.
E. 4 L normal saline.
ANSWER: A
108. All the following are true about liver anatomy except
A. Cantlie’s line extends roughly between the gallbladder fossa and the inferior vena cava.
B. The right lobe accounts around two thirds of the liver size.
C. The Caudate lobe lies posterior to the level of the porta hepatis.
D. The anterior branch of the right portal vein usually supplies liver segments V and VI.
E. The right hepatic vein drains venous tributaries from segments V, VI, VII and VIII.
ANSWER: D
109. All the following are true regarding the pancreatic ducts except
A. The duct of Santorini embryologically comes from the dorsal pancreatic bud.
B. The ventral bud forms most of the pancreas.
C. In most people most of the pancreas is drained via the major ampulla.
D. In most people the two pancreatic ducts are communicated.
E. The ampulla of Vator located between the proximal two thirds and the distal one third of the second part of
duodenum.
Answer: B
110. Which of the following does not pass through the diaphragmatic openings?
A. Aorta
B. Inferior Vena Cava
C. Esophagus
D. Left gastric artery
E. Left phrenic nerve
ANSWER: D
111. 10. In a total thyroidectomy for advanced papillary cancer, you notice the tumor on the right is adherent to
a nerve in the tracheo-esophageal groove. This nerve injury or resection will result in:
A. Loss of high-pitched voice
B. Stridor with need for tracheostomy
C. Unilateral right vocal cord paralysis
D. Loss of sensation above the vocal cords.
E. Loss of sensation in the skin of the right muscular triangle
ANSWER: C
112. A patient asks you when he can allow water to touch his closed surgical wound. Which of the following is
true pertaining to wound healing& external environment?
A. One week, as epithelialization and granulation tissue formation need to occur to protect the wound further,
even with meticulous closure
B. 24 hours, as meticulous closure with epithelialization seals the wound from external environment, in the
absence of excessive tension on wound
C. Two weeks, as acellular collagen starts forming and maturation phase occurs
D. Five days, as epithelialization requires this long, and collagen formation begins then
E. Three weeks to allow collagen type III, which is present in normal skin, to fully form
ANSWER: B
114. Which of the sutures listed below would be most suitable for suturing the femoral artery following an
embolectomy?
A. 5/0 polyglactin
B. 5/0 polypropylene
C. 5/0 silk
D. 5/0 polyglycolic acid
E. 5/0 polyester
ANSWER: B
115. A24-year-old woman is scheduled for an elective cholecystectomy. The best method of identifying a
potential bleeding is which of the following?
A. Platelet count
B. A complete history and physical examination
C. Bleeding time
D. Lee-White clotting time
E. Prothrombin time (PT)
ANSWER: B
116. A 60-year-old woman who underwent a mastectomy for breast cancer 2 years earlier presents to the
emergency department with headache, backache, and frequent vomiting. She is extremely thirsty and
stuporous. Which test is most likely to identify the cause?
A. CT scan of the head
B. X-ray of spine
C. Serum sodium determination
D. Serum calcium determination
E. Serum glucose determination
ANSWER: D
117. A 43-year-old woman with von Willebrand’s disease is scheduled for cholecystectomy. It can be stated
that preoperative evaluation will reveal which of the following?
A. Normal bleeding time, PT, and PTT
B. Platelet aggregate with restocetin
C. Increased bleeding time and PTT, and normal PT
D. Increased bleeding time and PT, and normal PTT
E. Increased bleeding time, and normal PT and PTT
ANSWER: C
118. A 64-year-old woman undergoing radical gastrectomy under general anesthesia is transfused with 2 U of
packed RBCs. A hemolytic transfusion reaction during anesthesia will be characterized by which of the
following?
A. Shaking chills and muscle spasms
B. Fever and oliguria
C. Hyperpyrexia and hypotension
D. Tachycardia and cyanosis
E. Bleeding and hypotension
ANSWER: E
119. After undergoing subtotal gastrectomy for carcinoma of the stomach, a 64-year-old woman is receiving
peripheral parenteral nutrition. To increase calories by the peripheral route, what should be prescribed?
A. D5W in normal saline
B. Multivitamin infusion
C. D25W (25% dextrose in water)
D. Soybean oil (intra lipid)
E. Lactulose
ANSWER: D
122. All of the following are true regarding human body fluids except
A. Water constitutes about 60% of the weight of an average adult.
B. Most of body water presents intracellular.
C. An average adult, about 75 kg weight, needs about 1400cc of water daily.
D. Urine constitutes the major source of water loss.
E. Interstitial water is much more than intravascular water.
ANSWER: C
123. In response to trauma, the serum levels of all of the following hormones increase except
A. T3.
B. Adrenalin.
C. Insulin.
D. Growth hormone.
E. Cortisol.
ANSWER: C
124. All of the following increase the amount of insensible water loss except.
A. Dry weather.
B. Fever.
C. Diarrhea.
D. Tracheotomy.
E. Hot weather.
ANSWER: C
127. All the following are true regarding respiratory alkalosis except
A. Caused by increase respiratory CO2 washing.
B. Kidneys compensate rapidly for the condition.
C. Compensation never overcorrect the pH.
D. Rebreathing mask can be used in treatment.
E. May be associated by carpopedal spasm.
ANSWER: B
130. Which of the following blood products carries the highest risk of transmission of bacterial infection?
A. Packed RBCs.
B. Fresh frozen plasma.
C. Platelets concentrates.
D. Cryoprecipitate.
E. Whole blood.
ANSWER: C
131. Which of the following is an advantage of transjugular central venous line over a central venous line
inserted through a peripheral vein?
A. We can give higher concentration of glucose solutions.
B. We can keep it for a longer period.
C. Less risk of pneumothorax.
D. Less risk of arrythmia.
E. Less risk of air embolism.
ANSWER: B
132. In the management of a patient who has massive trauma and hypotension, all of the following should be
done except
A. Stop hemorrhage.
B. Correct coagulopathy.
C. Elevate systolic pressure above 110mmHg.
D. Control sepsis or major contamination.
E. Give blood.
ANSWER: C
133. In comparison between healing by primary and secondary intentions for a wound involving skin and
subcutaneous fat, healing by primary intension has
A. More inflammatory response.
B. Less tensile strength.
C. Less chance to develop abscess.
D. Less scar.
E. More granulation tissue formation.
ANSWER: D
134. All of the following are true about polypropylene sutures except
A. It is monofilament.
B. It preserves its tensile strength for many years.
C. It is non absorbable.
D. It develops high tissue reaction.
E. It has high memory.
ANSWER: D
135. All of the following increase the risk of wound infection except
A. Obesity.
B. Jaundice.
C. Uremia.
D. Chemotherapy.
E. Pathing at the first postoperative day.
ANSWER: E
136. Which of the following sutures carries a higher risk of wound infection?
A. Silk.
B. Polypropylene.
C. Nylon.
D. Polyglactin.
E. Polydioxanone.
ANSWER: A
138. All of the following are steps done in processing a specimen for histological examination except
A. Fixation in formalin for one day.
B. Make representative tissue blocks.
C. Place blocks in paraffin wax.
D. Make thin slices on glass slides.
E. Stain slides with Indian ink.
ANSWER: E
139. All of the following are of the first step of pain stepladder booklet advised by WHO to control chronic pain
caused by malignancy, except
A. Tramadol.
B. Aspirin.
C. Paracetamol.
D. Nonsteroidal anti-inflammatory drugs.
E. Tricyclic antidepressants.
ANSWER: A
140. Measures to reduce the risk of venous thromboembolism include all of the following except
A. Prevent preoperative dehydration.
B. Start to apply graduated compressing lower limbs stockings immediately after surgery.
C. Apply intraoperative foot impulse device.
D. Use intraoperative pneumatic compressing device.
E. Start prophylactic low molecular weight heparin preoperatively.
ANSWER: B
141. All of the following combinations between an ulcer’s edge and pathology are true except
A. Sloping edge: healing ulcer.
B. Undermined edge: bed sore.
C. Rolled edge: basal cell carcinoma.
D. Everted edge: tuberculosis.
E. Punched-out edge: syphilis.
ANSWER: D
142. All of the following are proven examples of congenital sinuses except
A. Preauricular.
B. Umbilical.
C. Pilonidal.
D. Urachus.
E. Coccygeal.
ANSWER: C
144. All of the following should be checked before induction of anesthesia in a patient planed for paraumbilical
hernia repair except
A. Confirm the patient’s name.
B. Confirm the diagnosis.
C. Check the consent.
D. Mark the site of the procedure.
E. Ask for known allergies.
ANSWER: D
147. All the following are true about venous ulcers except
A. Associated with varicose veins.
B. Usually on the lateral aspect of the leg.
C. Usually shallow.
D. The affected limb is usually warm.
E. More common in women.
ANSWER: B
148. Which of the following organs creates the least immunological response in deceased donor organ
transplantation?
A. Liver.
B. Kidney.
C. Cornea.
D. Pancreas.
E. Small bowel.
ANSWER: C
149. All the following are true about trichilemmal cyst except
A. More common in the scalp.
B. Has a distinct odor if ruptured.
C. Has a thin wall.
D. The wall is a complete mature epidermis with granular layer.
E. If causes symptoms, the entire cyst is removed.
ANSWER: D
151. All the following are true about laryngeal mask airway except
A. It sits in the oropharynx.
B. It is passed to its site under vision.
C. It has a cuff which is inflated to push the soft tissue away from the laryngeal inlet.
D. It does not pass through the vocal cords.
E. It does not fully protect against aspiration.
ANSWER: B
152. Which of the following statements is true regarding the radiographic appearance of small bowel
obstruction?
A. Gas within the small bowel is distinguished from gas within the colon by luminal lines perpendicular
to the bowel wall. The small bowel lines partially cross the lumen, whereas the colonic lines
completely cross the lumen.
B. Ileus may be difficult to distinguish from small bowel obstruction, because both conditions can
produce gaseous distention of the bowel with air-fluid levels.
C. The “string of pearls” sign refers to a series of radiolucent images in the small bowel
representing the gallstones of gallstone ileus.
D. A gasless abdomen seen on plain films rules out small bowel obstruction.
E. The distinction between complete and partial small bowel obstructions during the early stages is
made by assessing the colonic gas pattern.
Answer: B
153. Which of the following statements regarding short bowel syndrome (SBS) is true?
A. Resection of up to 70% of the bowel can be tolerated if the terminal ileum and ileocecal valve are
preserved.
B. Diarrhea is best controlled by the administration of medium-chain triglycerides.
C. The administration of oral bile salts is of central importance in controlling steatorrhea.
D. Vagotomy and pyloroplasty and reversal of a segment of bowel are the two most important operations for
the early management of SBS.
E. Relative gastric hyposecretion, with increased intestinal pH in conjunction with interruption of the
enterohepatic bile salt circulation, is the cause of steatorrhea.
Answer: A
154. Which of the following surgical techniques lead to improved wound healing, except?
A. Atraumatic handling of tissue
B. Deep sutures are best placed only into collagen laden structures
C. Approximation of underlying fatty tissue to obliterate dead space
D. Allowing the patient to wash or shower one or two days after surgery
E. Meticulous hemostasis
Answer: C
155. The body's response to hypothermia includes all of the following, except
A. Decreased platelet function
B. Decrease cardiac output
C. Tachycardia
D. Cardiac arrhythmia
E. Increased risk of wound infection
Answer: C
158. A 65-year-old woman is 3 days status post laparoscopic left colectomy for diverticulitis, and acute short of
breath develops. On clinical assessment your suspicion is that the patient has pneumonia but you have a
low to moderate concern for pulmonary embolism. The next test of choice is:
A. CT angiography
B. Nuclear ventilation/ perfusion scan
C. D-Dimer rapid enzyme-linked immunosorbent assay
D. Bilateral leg vein duplex ultrasound scan
E. Pulmonary arteriogram
Answer: C
159. An obese 50-year-old man who has had a renal transplant undergoes left direct hernia repair with mesh
under local anesthesia. Medications .include tacrolimus (Prograf) and prednisone. A nasal swab is positive
for methicillin-resistant Staphylococcus aureus (MRSA). Which of the following statements is TRUE?
A. His wound classification is clean
B. He should receive oral antibiotics for 48 hours postoperatively
C. His risk of surgical site infection is less than 1 %
D. Mesh is contraindicated in this patient
E. The wound should be irrigated with vancomycin
Answer: A
160. Which of the following statements about combining neuraxial (epidural) analgesia to improve
pulmonary function after abdominal operation with chemical prophylaxis to prevent postoperative
venous thromboembolic events is TRUE?
A. Subcutaneous unfractionated heparin (UFH) is contraindicated with epidural analgesia
B. Subcutaneous low molecular weight heparin (LMWH) administered every 12 hours is contraindicated
with epidural analgesia
C. Subcutaneous LMWH administered every 24 hours is contraindicated with epidural analgesia
D. UFH must be held for 36 hours prior to epidural catheter removal
E. LMWH must be held for 36 hours prior to epidural catheter removal
Answer: B
161. Which of the following is always improved after gastric bypass surgery
A. DM
B. Hypertension
C. GERD
D. Weight loss
E. Osteoporosis
Answer: C
163. What is the management or persistent, localized, MALT lymphoma following repeatedly failed efforts at H.
pylori eradication therapy?
A. Radiation therapy
B. Surgical resection
C. Chemotherapy
D. Rituximab
E. Bevacicumab
Answer: A
164. Which of the following statements about branchial cleft remnants is NOT true?
A. Arise most commonly from the third branchial arch
B. Fistula formation common at anterior border of the stemocleidomastoid muscle
C. Often present as recurrent neck infections
D. Cyst tends to present later than fistula
E. Surgical excision is treatment of choice
Answer: A
165. Increased perioperative mortality after lower extremity amputation is associated with
A. Race
B. Sex
C. Level of amputation
D. Smoking history
E. Type of anesthetic
Answer: C
166. Which of the following is a poor prognostic indicator in melanoma patients?
A. Extremity location
B. Low Breslow thickness
C. Female gender
D. Elevated LDH
E. Younger age
Answer: D
167. An 86-year-old female presents to your office with a new 1.8-cm raised pearly nodule with surface
telangiectasias on her cheek. Which of the following would be the best option in her management?
A. Surgical excision with 1 cm margins
B. Cryotherapy
C. Mohs microsurgery
D. Radiation therapy
E. Topical imiquimod
Answer: C
168. A 47-year-old female presents with a several months history of a slowly growing painless mass in her
right thigh. On physical exam, she is noted to have a 5 cm firm, non-tender mass in the right lateral mid-
thigh. Which of the following is true?
A. Routine imaging of the mass should include a PET scan.
B. Incisional biopsy should be routinely performed in order to confirm the diagnosis and identify the
histological subtype.
C. The most common histologic subtypes for extremity sarcomas include liposarcoma and malignant fibrous
histiocytoma.
D. Staging includes an abdominal and pelvic CT scan to evaluate or metastatic disease in this lesion.
E. Multidisciplinary evaluation is not needed for simple, early-stage sarcomas.
Answer: C
169. A 68-year-old male with a history of hypertension and hypercholesterolemia presents to his primary care
physician’s office with a chief complaint of worsening epigastric pain and weakness. The pain is improved
with oral intake, especially milk-based products. The patient has been treating his pain with naproxen. In
the office, the patient is non-toxic with normal vital signs. His physical examination reveals mild epigastric
tenderness with deep palpation. Serum hemoglobin was 8.3 g/dL. Fecal occult blood testing was positive.
The patient underwent colonoscopy, which was normal. Esophagogastroduodenoscopy (EGD) revealed A
2.5 cm ulcerated lesion with elevated, irregular Borders 5 cm distal to the gastroesophageal junction. Final
pathology reveals a poorly differentiated adenocarcinoma. The most sensitive preoperative examination to
determine T and N stage is:
A. Positron emission tomography (PET) scan
B. Endoscopic ultrasound (EUS)
C. Magnetic resonance imaging (MRI) with gadolinium
D. Diagnostic laparoscopy
E. Triple-phase helical computed tomography (CT) scan
Answer: B
170. A 78-year-old female with arthritis and asthma presents to the emergency room with an acute onset of
epigastric pain a couple of hours ago. Her pulse is 104, her blood pressure is unchanged from her baseline
of 110/74 mm Hg, and her temperature is 98.2 Fahrenheit. Her medications include occasional Naproxen
and prednisone for occasional exacerbations of her asthma. On physical exam, her abdomen
demonstrates significant epigastric tenderness with rebound. She has a mild leukocytosis o 12.5 cells/mcL.
Her acute abdominal series demonstrates a small amount of free air. What is the most reasonable current
treatment option for this patient as the next step?
A. Laparoscopic highly selective vagotomy without resuscitation
B. Nasogastric tube insertion, cessation of all oral feeds, & intravenous fluid initiation or the next 24 hours
C. Open truncal vagotomy with pyloroplasty
D. Open Graham patch with parietal cell vagotomy despite laparoscopic experience and resources
E. Emergent anterior seromyotomy
Answer: B
171. A 35-year-old female with past medical history of hypertension, type II diabetes mellitus, and morbid
obesity recently underwent a gastric bypass surgery 8 months ago without complication. On her last visit
office visit 2 months ago, no complications were noted and weight was found to be appropriate. She is
currently mourning the loss of her grandmother who passed away 2 weeks ago. Today, her daughter
noticed that her gait was very unsteady and she seemed to be confused. She also mentioned that she had
very poor appetite and was very unclear if she has been taking her medications. There is no history of
alcohol intake. She was taken to the emergency department for further evaluation. What is the vitamin
deficiency most likely associated with the above presentation?
A. Vitamin D
B. Folate
C. Vitamin A
D. Thiamine
E. Cobalamin
Answer: D
172. A 66-year-old Japanese female is re erred by her primary physician for long-standing biliary colic
symptoms. She describes 10 to15 years of intermittent right upper quadrant (RUQ) pain with nausea that
typically resolves after 1 to 2 hours. She went to the emergency room once 6 years ago and had an
ultrasound that showed gallstones. Her medical history is significant or hypertension and osteoporosis. Her
vitals and exam are unremarkable. A repeat RUQ ultrasound ordered by her primary care physician now
shows a large 3 cm gallstone, as well as a fixed mass in the fundus, 2 cm in diameter that appears to
originate from the gallbladder wall. The immediate surrounding gallbladder wall is thickened to 8 to 11 mm.
A CBC, basic chemistry, and liver function tests are all within normal limits. Which of the following
radiographic findings is associated with the highest incidence of gallbladder carcinoma?
A. Pancreaticobiliary maljunction (PBM) without biliary dilatation identified on magnetic resonance
cholangiopancreatography (MRCP).
B. Strongly enhancing thick inner layer and a weakly enhancing or nonenhancing outer layer of the
gallbladder wall on the portal phase of amulti-detector CT scan (MDCT).
C. Gallbladder polyp 10–20 mm on ultrasound.
D. Gallbladder wall calcifications; “porcelain gall- bladder”.
E. Asymptomatic gallstone greater than 3 cm in size.
Answer: D
173. A 67-year-old female presents with sharp, burning right upper quadrant abdominal pain that radiates to
her back. The pain awakens her from sleep. She has experienced nausea and three episodes of emesis.
She reports burning substernal and Epigastric pain. She denies any fevers or chills. She is
hemodynamically normal and afebrile. Laboratory workup reveals a normal hepatic function panel, a
normal basic chemistry, and a normal white blood cell count. Her abdominal exam is remarkable for mild
tenderness in the right upper quadrant without peritoneal signs. After three hours, her pain resolves. What
is the next best test in this patient’s scenario?
A. Plain films of the abdomen
B. Cholecystokinin stimulated cholescintigraphy
C. Right upper quadrant ultrasound
D. Computed tomography (CT) of the abdomen
E. Esophagogastroduodenoscopy (EGD)
Answer: C
174. A 70-year-old male presents to the emergency department with altered mental status. Family reports he
was complaining of right upper quadrant abdominal pain prior to becoming altered mentally. On arrival, he
is found to have a temperature of 102.5°F, heart rate of 112, and systolic blood pressure of 80 despite 2
liters of crystalloid infusion. On exam, he is visibly jaundiced with tenderness in the right upper quadrant.
He has 3 out of 4 systemic inflammatory response syndrome (SIRS) criteria. He is started on
piperacillin/tazobactam and admitted to the intensive care unit or invasive monitoring and vasopressor
support. Regarding the management of septic cholangitis caused by choledocholithiasis, which of the
following is correct?
A. Urgent biliary tract decompression via endoscopic retrograde cholangiopancreatography (ERCP) is
successful 60% of the time.
B. Percutaneous transhepatic cholangiography (PTC) is feasible for stone extraction and stent placement.
C. ERCP with a sphincterotomy is equivalent to cholecystectomy for reducing recurrence rates.
D. Should ERCP and PTC fail or are not feasible, operative choledochotomy and T-tube placement should be
avoided because of the risk of surgery.
E. Broad spectrum antibiotic therapy alone will generally provide adequate treatment.
Answer: B
175. A 60-year-old male presents with 2 months history of rectal bleeding associated with anorexia and a 10 lb
weight loss. He underwent a diagnostic colonoscopy revealing an ulcerated and circumferential right cecal
mass; biopsy returns as invasive adenocarcinoma. Staging CT scan of chest/abdomen/pelvis shows
bilobar and peripheral liver masses ranging from 2 to 3 cm in diameter with a total of 3 masses, no regional
or para-aortic adenopathy, and no other distant sites of disease. Percutaneous biopsy of a right lobar liver
mass reveals metastatic adenocarcinoma, consistent with the colon as the primary site of malignancy. The
patient has hypertension and hypercholesterolemia. He has no family history of colorectal cancer. His
functional status is good. The best initial management in this patient is:
A. Neoadjuvant chemotherapy
B. Concomitant right hemicolectomy and live resection
C. Right colectomy then chemotherapy
D. Radio frequency ablation o liver tumors
E. Hepatic Intra-arterial chemotherapy
Answer: B
176. A 60-year-old male with no significant medical history except a long history of smoking presents with
painless jaundice of 1 week duration. He denies any symptoms of obstruction, weight loss, or fatigue. Initial
labs show mild elevation in transaminases to less than 1.5 times normal values and a bilirubin of 8 mg/dL.
A right upper quadrant (RUQ) ultrasound was obtained that shows dilated intra and extra-hepatic bile ducts
and a dilated gallbladder without stones. The pancreas is not well visualized secondary to overlying bowel
gas. What would be the next best step in the evaluation of this patient?
A. Magnetic Resonance Cholangiopancreatigram (MRCP).
B. Thin slice CT scan with arterial and venous phases.
C. Whole body Positron Emission Tomography (PET) scan.
D. Endoscopic Retrograde Cholangiopancreatigraphy (ERCP).
E. CA19-9 laboratory specimen.
Answer: B
177. A 50-year-old otherwise healthy male presents to the emergency room with nausea, vomiting, and
abdominal pain. He has had a one month history of chronic intermittent abdominal pain. He has no prior
history of abdominal surgeries and had a normal colonoscopy earlier this year. His vital signs are within
normal limits. On exam, His abdomen is soft, non-distended, with mild diffuse abdominal tenderness. There
are no hernias present. His laboratory exam is unremarkable. He undergoes an abdominal CT scan which
demonstrates an area of intussusception in the distal small bowel with thickening and calcification of the
adjacent mesentery. What is the best next step in diagnosis?
A. Small bowel follow through
B. Abdominal MRI
C. Capsule Endoscopy
D. Barium Enema
E. Diagnostic Laparoscopy
Answer: E
178. A 67-year-old male with a history of hypertension, atrial fibrillation, and obesity presents to the emergency
department with acute onset abdominal pain, nausea, vomiting, and hematochezia. Previous operations
include a laparoscopic appendectomy. On physical exam the patient appears to be in significant distress
due to pain; however, his abdomen is soft, non-tender, and non-distended. Laboratory analysis is
remarkable or a white blood cell count o 21 × 109/mL and a lactate o 3.5 mmol/L. Abdominal radiography is
unremarkable. C angiography is significant or absence of flow distal to the origin of the superior mesenteric
artery (SMA).What is the likely etiology or this SMA occlusion?
A. Splanchnic vasoconstriction
B. Embolic occlusion o the mesenteric circulation
C. Acute thrombosis o the mesenteric circulation
D. Hypercoagulable state
E. Mesenteric venous thrombosis (MV )
Answer: B
179. Regarding the clinical manifestations of acute appendicitis diagnosis, which of the following is correct?
A. The location of the appendiceal tip has little to do with determining the presenting symptoms.
B. Only 25% of adult patients with acute appendicitis will present with a “classic” history.
C. A clinical diagnosis of acute appendicitis is more accurate in men than women.
D. The appendiceal obstruction is not the hypothesized pathogenesis of acute appendicitis.
E. Delay in diagnosis and/or surgical intervention is the most common cause of complicated (gangrenous or
per orated) appendicitis rather than patient delay in seeking medical attention.
Answer: C
180. Regarding possibilities that may occur during a laparoscopic appendectomy, which of the following is
correct?
A. If the appendix appears normal, a diagnostic laparoscopy should be performed to look or other potential
causes of the patient’s right lower quadrant pain. The appendix should NO be removed in such cases.
B. If a sub-centimeter mass is identified at the appendiceal tip, one should perform an ileocecectomy.
C. If an enterotomy occurs during trocar placement, one should perform a bowel resection and not attempt
primary repair.
D. If a free rupture of the appendix with fecal contamination of the peritoneal cavity has occurred, one should
do a washout only at this operation and manage the patient post-operatively with antibiotics.
E. If one encounters a perforated appendix, leaving a drain has not been shown to reduce the rate of abscess
formation.
Answer: E
181. Which of the following is true regarding the development of cancer in Crohn’s disease?
A. There is a higher rate of a Crohn’s associated malignancy in a fistula tract compared to a stricture.
B. The risk of colon cancer in Crohn’s disease is no greater than the general population.
C. Surveillance colonoscopy is not indicated in a patient with an ileosigmoid fistula.
D. Colon cancer in the setting of Crohn’s disease should be managed similar to the general population.
E. Subtotal colectomy with ileorectal anastomosis has better outcomes in Crohn’s disease than a segmental
colectomy.
Answer: D
182. A surgical intervention should be offered or which of the following presentations o Crohn’s disease?
A. Enteroenteric fistulas with mild symptoms.
B. Medically managed disease but on two medications.
C. New inflammatory changes of the terminal ileum on radiographic findings.
D. Ileosigmoid fistula and associated diarrhea and malnutrition.
E. Intra-abdominal abscess amenable to CT–guided percutaneous drainage.
Answer: D
183. A 47-year-old obese man presents to the emergency room with a two day history of sharp, constant, left
lower quadrant pain that is worse with straining and is associated with subjective fevers. This is the
patient’s first episode of pain. His vital signs are normal and he is in no distress. You are able to elicit
moderate tenderness in the le lower quadrant on exam, and a complete blood count shows a mild
leukocytosis o 13,000/μL. 2. Regarding the treatment of acute, uncomplicated, sigmoid diverticulitis in a
reliable patient, what is the most appropriate treatment plan?
A. Discharge with pain medication.
B. Discharge with oral (PO) antibiotics and close follow up.
C. Admission, IV antibiotics, and bowel rest.
D. Urgent exploratory laparotomy and resection o the involved colon.
E. Discharge with PO antibiotics and planned laparoscopic resection in 6 to 8 weeks.
Answer: B
184. A 48-year-old white man presents to your office with a history of seven months of intermittent painless
blood per rectum with bowel movements, which was diagnosed as internal hemorrhoids by his primary care
provider. He has no other medical problems and has an unknown family history because he was adopted
as an in ant. On your digital and anoscopic evaluations, you find scant bloody mucous, no pathologic
hemorrhoids, no other anal diseases, and the visualized area o rectal mucosa is normal. What is the next
best step in his evaluation?
A. Immunohistochemical fecal occult blood test
B. Guaiac fecal occult blood test
C. Colonoscopy
D. Computed tomographic (CT) colonography
E. Flexible sigmoidoscopy
Answer: C
186. Pathology reveals lobular carcinoma in situ (LCIS). The patient now seeks advice on her overall risk of
cancer. Which of the following is true?
A. LCIS has a 40% life-time risk of development into lobular carcinoma.
B. LCIS is not considered a precursor itself, but has an increased risk of malignancy in either breast.
C. Ductal carcinoma in situ (DCIS) is not considered a precursor to malignancy.
D. LCIS and DCIS are both precursors to malignancy.
E. LCIS is the most common type o in-situ breast lesion.
Answer: B
187. With regard to adjuvant tamoxifen for DCIS, which of the following statements is true?
A. Tamoxifen (TMX) improves both local recurrence and survival rates in DCIS. B. MX has no significant
effect on survival in invasive breast cancer.
B. Side effects of TMX includes an increased risk of ovarian cancer.
C. Tamoxifen is equally effective in ER+ lesions as in ER− lesions.
D. In women undergoing breast conservation therapy or DCIS, TMX decreases invasive and non-invasive
breast cancer events or both the ipsilateral and contralateral breast
Answer: E
188. What is the underlying pathophysiology in inflammatory breast cancer contributing to the skin changes
that are noted on clinical exam?
A. Infection (mastitis/abscess)
B. Tumor emboli obstructing the dermal lymphatic vessels
C. Localized dermatitis
D. Congestive heart failure causing skin edema
E. Concurrent non-Hodgkin Lymphoma
Answer: B
189. A 23-year-old Pacific Islander female is referred to you for evaluation a left neck mass. She rarely
receives any medical care. She has noticed the neck mass or the past 6 months. The lump is increasing in
size with time. She states that otherwise, she feels normal. On physical exam, she has a palpable lump on
her left thyroid gland as well as left neck lymphadenopathy. She is very thin with long limbs and round, firm
lumps in her lips. You suspect she has multiple endocrine neoplasia (MEN) 2B syndrome. What potential
abnormalities would you suspect and work up?
A. Papillary thyroid cancer
B. Pituitary adenoma
C. Pheochromocytoma
D. Parathyroid hyperplasia
E. Hirschsprungs
Answer: C
190. A 4-year-old girl is brought in to see her primary care provider (PCP) by her parents. They have noticed a
small mass on the anterior portion of her neck, but recently started noticing that it was spontaneously
draining and has become erythematous. Her PCP has the girl stick out her tongue and notices that the
mass moves with tongue protrusion. The physician correctly identifies that this is likely an infected
thyroglossal duct cyst. She appropriately places the child on antibiotics, covering for oral flora, and then
refers the child to surgery or operative intervention, delayed until the infection clears. What is the most
appropriate treatment or a thyroglossal duct cyst?
A. Surgical resection o the cyst, taking care to only remove the cyst and not disturb any of the critical
surrounding neck structures
B. Resection of the thyroglossal duct and the complete hyoid bone
C. Removal of the cyst en bloc, excising a core of tissue around the tract to the base o the tongue, and
including the central 1/3rd of the hyoid bone
D. Total thyroidectomy
Answer: C
191. What is the usual acid-base dysfunction associated with infantile hypertrophic pyloric stenosis (IHPS)?
A. Hyperchloremic, hyperkalemic metabolic alkalosis with alkalotic urine
B. Hypochloremic, hypokalemic metabolic alkalosis with alkalotic urine
C. Hyperchloremic, hypokalemic metabolic acidosis with alkalotic urine
D. Hypochloremic, hypokalemic metabolic alkalosis with aciduria
E. Hyochloremic, hyperkalemic metabolic alkalosis with aciduria
Answer: D
192. Regarding the pathophysiology of pediatric abdominal wall defects, which of the following is correct?
A. Polyhydramnios and intestinal atresia are not associated with omphalocele or gastroschisis.
B. Gastroschisis is commonly associated with cardiac and chromosomal anomalies.
C. A giant omphalocele is more likely to be associated with fetal aneuploidy than gastroschisis.
D. Intestinal non-rotation is not associated with gastroschisis or omphalocele.
E. Herniated liver indicates the defect is most likely an omphalocele.
Answer: E
193. Concerning base skull fracture of the anterior fossa all of the following is true, except:
A. Epistaxis.
B. CSF rhinorrhea.
C. Blindness, anosmia
D. Raccon eyes.
E. Battle sign.
Answer: E
194. All of the following are hard signs of vascular injury except :
A. Expanding hematoma
B. Weak distal pulse
C. Limb ischemia
D. Active external hemorrhage
E. Abnormal ABI (less than 0.9)
Answer: B
2nd year:
1. Which of the following statements is true about the liver enzymes ALT and AST?
A. ALT is more specific for liver disease than AST.
B. Both participate in glycolysis.
C. They are usually severely elevated in cases of liver cirrhosis.
D. In alcoholic liver disease, an ALT:AST ratio more than 2 is common.
E. In fulminant hepatitis, ALT and AST levels are usually moderately elevated.
ANSWER: A
3. All the following about the vascular system of the gallbladder are true except
A. The cystic artery is usually a branch of the right hepatic artery.
B. When the cystic artery reaches the neck of the gallbladder it divides into anterior and posterior branches.
C. Usually, a single cystic vein drains the gallbladder to the portal vein.
D. Frequently, a visible lymph node overlies the cystic artery near its end.
E. The nerves of the gallbladder come through the celiac plexus.
ANSWER: C
13. Which is the most common cause of peptic ulcer disease (PUD) of the following?
A. Smoking
B. Non steroidal anti inflammatory drugs
C. Zollinger-Ellison syndrome
D. Ethanol excess
E. Family history of PUD
Answer: B
14. With regard to bipolar and monopolar devices -
A. Both use direct current (DC).
B. Both use alternating current (AC).
C. Bipolar uses AC, while monopolar uses DC.
D. Bipolar uses DC, while monopolar uses AC.
E. Neither requires an electrosurgical unit (ESU). Coagulation
ANSWER: B
15. Which of the following has been shown to reduce the incidence of acquired central venous line sepsis?
A. Administration of intravenous (IV) vancomycin prophylaxis at the time of line insertion
B. Utilizing the femoral vein site of insertion
C. Using a multiple lumen catheter
D. Daily review and removal of the catheter at the earliest opportunity
E. Changing the lines every seven days
ANSWER: D
17. A 20 years old male patient, presented complaining of abdominal pain and vomiting for 24 hours due to
intestinal obstruction, he is thirsty with dry mouth, mucous membranes and axillae, he has mild tachycardia
but normal supine blood pressure, his weight is 80kg. Which of the following parenteral fluid therapy
formulas may be more suitable for him in the first 24 hours?
A. 8 L glucose saline 0.45%.
B. 1 L normal saline and 2 L glucose water 5%.
C. 2 L normal saline and 1 L glucose water 5%.
D. 4 L glucose saline 0.18%.
E. 4 L normal saline.
ANSWER: A
18. All the following are true about liver anatomy except
A. Cantlie’s line extends roughly between the gallbladder fossa and the inferior vena cava.
B. The right lobe accounts around two thirds of the liver size.
C. The Caudate lobe lies posterior to the level of the porta hepatis.
D. The anterior branch of the right portal vein usually supplies liver segments V and VI.
E. The right hepatic vein drains venous tributaries from segments V, VI, VII and VIII.
ANSWER: D
19. All the following are true regarding the pancreatic ducts except
A. The duct of Santorini embryologically comes from the dorsal pancreatic bud.
B. The ventral bud forms most of the pancreas.
C. In most people most of the pancreas is drained via the major ampulla.
D. In most people the two pancreatic ducts are communicated.
E. The ampulla of Vator located between the proximal two thirds and the distal one third of the second part of
duodenum.
Answer: B
20. Which of the following does not pass through the diaphragmatic openings?
A. Aorta
B. Inferior Vena Cava
C. Esophagus
D. Left gastric artery
E. Left phrenic nerve
ANSWER: D
21. In a total thyroidectomy for advanced papillary cancer, you notice the tumor on the right is adherent to a
nerve in the tracheo-esophageal groove. This nerve injury or resection will result in:
A. Loss of high-pitched voice
B. Stridor with need for tracheostomy
C. Unilateral right vocal cord paralysis
D. Loss of sensation above the vocal cords.
E. Loss of sensation in the skin of the right muscular triangle
ANSWER: C
23. Which of the sutures listed below would be most suitable for suturing the femoral artery following an
embolectomy?
A. 5/0 polyglactin
B. 5/0 polypropylene
C. 5/0 silk
D. 5/0 polyglycolic acid
E. 5/0 polyester
ANSWER: B
24. A 60-year-old woman who underwent a mastectomy for breast cancer 2 years earlier presents to the
emergency department with headache, backache, and frequent vomiting. She is extremely thirsty and
stuporous. Which test is most likely to identify the cause?
A. CT scan of the head
B. X-ray of spine
C. Serum sodium determination
D. Serum calcium determination
E. Serum glucose determination
ANSWER: D
25. A 43-year-old woman with von Willebrand’s disease is scheduled for cholecystectomy. It can be stated that
preoperative evaluation will reveal which of the following?
A. Normal bleeding time, PT, and PTT
B. Platelet aggregate with restocetin
C. Increased bleeding time and PTT, and normal PT
D. Increased bleeding time and PT, and normal PTT
E. Increased bleeding time, and normal PT and PTT
ANSWER: C
26. A 64-year-old woman undergoing radical gastrectomy under general anesthesia is transfused with 2 U of
packed RBCs. A hemolytic transfusion reaction during anesthesia will be characterized by which of the
following?
A. Shaking chills and muscle spasms
B. Fever and oliguria
C. Hyperpyrexia and hypotension
D. Tachycardia and cyanosis
E. Bleeding and hypotension
ANSWER: E
28. All of the following are true regarding human body fluids except
A. Water constitutes about 60% of the weight of an average adult.
B. Most of body water presents intracellular.
C. An average adult, about 75 kg weight, needs about 1400cc of water daily.
D. Urine constitutes the major source of water loss.
E. Interstitial water is much more than intravascular water.
ANSWER: C
29. In response to trauma, the serum levels of all of the following hormones increase except
A. T3.
B. Adrenalin.
C. Insulin.
D. Growth hormone.
E. Cortisol.
ANSWER: C
30. All the following are true regarding respiratory alkalosis except
A. Caused by increase respiratory CO2 washing.
B. Kidneys compensate rapidly for the condition.
C. Compensation never overcorrect the pH.
D. Rebreathing mask can be used in treatment.
E. May be associated by carpopedal spasm.
ANSWER: B
31. In comparison between healing by primary and secondary intentions for a wound involving skin and
subcutaneous fat, healing by primary intension has
A. More inflammatory response.
B. Less tensile strength.
C. Less chance to develop abscess.
D. Less scar.
E. More granulation tissue formation.
ANSWER: D
32. All of the following are true about polypropylene sutures except
A. It is monofilament.
B. It preserves its tensile strength for many years.
C. It is non absorbable.
D. It develops high tissue reaction.
E. It has high memory.
ANSWER: D
33. All of the following increase the risk of wound infection except
A. Obesity.
B. Jaundice.
C. Uremia.
D. Chemotherapy.
E. Pathing at the first postoperative day.
ANSWER: E
35. All the following are true about venous ulcers except
A. Associated with varicose veins.
B. Usually on the lateral aspect of the leg.
C. Usually shallow.
D. The affected limb is usually warm.
E. More common in women.
ANSWER: B
36. All the following are true about laryngeal mask airway except
A. It sits in the oropharynx.
B. It is passed to its site under vision.
C. It has a cuff which is inflated to push the soft tissue away from the laryngeal inlet.
D. It does not pass through the vocal cords.
E. It does not fully protect against aspiration.
ANSWER: B
38. In clinical practice, cryoprecipitate is commonly used to supply the patient with coagulation factor
A. V
B. VII
C. VIII
D. IX
E. X
ANSWER: C
40. A 22 years old male patient presented with an infected surgical wound. His temperature is 38.2 degrees C,
pulse 100 per minute, blood pressure 120/75, respiratory rate 16/minute, WBCs 10.5 by 109 /L. His
condition is best termed
A. Systemic inflammatory distress syndrome without sepsis.
B. Sepsis.
C. Sever sepsis.
D. Stage 1 septic shock.
E. Multiorgan dysfunction.
ANSWER: B
42. A 30 years old male who was involved in a road traffic accident and underwent laparotomy and
splenectomy. In his second postoperative day he developed fever. This fever is most likely due to
A. Atelectasis.
B. Wound infection.
C. Post-splenectomy sepsis.
D. Urinary tract infection.
E. Deep vein thrombosis.
ANSWER: A
43. Which of the following fistulae has a higher chance to heal spontaneously?
A. Low fistula in ano.
B. Postoperative colo-cutaneous fistula with daily output of 70cc at the 14th day of diagnosis.
C. Biliary fistula with distal obstruction.
D. Ileal fistula caused by Crohn’s disease.
E. A fistula caused by a perforation in a colonic tumor which is not operated.
ANSWER: B
44. The best test to diagnose or exclude cancer in a 43 years old lady presented with breast mass is
A. Mammography
B. Breast ultrasound
C. Chest CT scan
D. Fine needle aspiration cytology
E. True-cut biopsy
ANSWER: E
45. All the following arteries are parts of the blood supply of the thyroid gland except
A. Thyroid ima
B. Superior thyroid arteries
C. Inferior thyroid arteries
D. Middle thyroid arteries
E. Branches of the thyro-cervical trunks
ANSWER: D
48. All the following are better in laparoscopic surgery than open laparotomy except
A. Less post-operative pain.
B. Better control of bleeding.
C. Better viewing of important structures in cholecystectomy.
D. Better exposure of subcostal regions.
E. Earlier return to work.
ANSWER: B
49. The circular end-to-end anastomosis (CEEA) stapler device is most valuable in
A. Anastomosing the small bowel together.
B. Anastomosing the lower esophagus to a R-en-Y jejunal loop.
C. Anastomosing the small bowel to the transverse colon.
D. Anastomosing the right and left colon.
E. Closing a tracheoesophageal fistula.
ANSWER: B
50. All the following suggest the possibility of inhalational injury in burn patients except
A. Swollen lips.
B. Change of voice.
C. Stridor.
D. Flam burn.
E. Burn happened in a closed space.
ANSWER: D
51. Low pitch sound after thyroidectomy is usually attributed to malfunction of:
A. Omohyoid muscle.
B. Sternohyoid muscle.
C. Cricothyroid muscle.
D. Thyrohyoid muscle.
E. Sternothyroid muscle.
ANSWER: C
52. Serum calcium level is usually elevated in all of the followings except:
A. Primary hyperparathyroidism.
B. Secondary hyperparathyroidism.
C. Tertiary hyperparathyroidism.
D. Hyperparathyroidism due to ectopic adenoma.
E. Vitamin D intoxication.
ANSWER: B
53. Unilateral bloody nipple discharge from one duct orifice is most commonly caused by:
A. Paget's disease of the nipple.
B. Carcinoma in situ.
C. Inflammatory carcinoma.
D. Subareolar mastitis.
E. Intraductal papilloma.
ANSWER: E
54. Which of the following has the highest risk for Colo-rectal carcinoma?
A. Ulcerative colitis.
B. Crohn’s disease.
C. Familial polyposis coli.
D. Diverticulosis.
E. Juvenile polyps.
ANSWER: C
55. A 35 years old male patient developed large amount of fresh bleeding per rectum 10 days post
hemorrhoidectomy. The most probable cause for his bleeding is
A. Sloughing of a vessel wall.
B. Uncontrolled bleeding missed at the time of surgery.
C. Clot dislodgment.
D. Injury by defecation.
E. Vasodilation.
ANSWER: A
56. Neoplastic cells have the ability to spread. One of the following is true:
A. Lymph node metastases permeate the sinusoids of the node and later spread throughout the subcapsular
space
B. Carcinoma in situ has no detectable invasion beyond the basement membrane
C. Lymphatic involvement is common with sarcomas
D. The metastatic process of malignancy is highly efficient, as once they shed into the bloodstream, they
produce metastasis almost always
E. Epithelial neoplasm spread only through hematogenous route
ANSWER: B
57. A 47-year-old woman presents to the emergency room after a motor vehicle accident, mildly lethargic and
confused, with normal airway and vitals. She complains of a headache, and has a frontal scalp hematoma.
After stabilizing her cervical spine and doing primary survey, she becomes more lethargic, moving only to
pain. Which of the following is the most likely to be present?
A. Subdural hematoma
B. Extradural hematoma
C. Cerebral contusion
D. Intracerebral hematoma
E. Diffuse axonal injury
ANSWER: B
58. If you want to review a reference paper addressing the best method of management yielding the best
outcome, which of the following has the best level of evidence?
A. A well-powered double-blinded randomized controlled trial
B. A case-series
C. A retrospective series of more than 1000 patients
D. Meta-analysis of retrospective series
E. A case-report
ANSWER: A
59. All of the following are gases used to provide pneumoperitoneum in laparoscopic surgery except?
A. Methane
B. Carbon dioxide
C. Helium
D. Nitrous oxide
E. Argon.
ANSWER: A
60. In which region of the GI tract is the longitudinal muscle of the muscularis arranged into distinct longitudinal
bundles?
A. Duodenum
B. Jejunum
C. Ileum
D. Colon
E. Stomach
ANSWER: D
61. All of the following conditions are associated with low cardiac output except
A. Anaphylactic shock.
B. Cardiac tamponade.
C. Tension pneumothorax.
D. Massive pulmonary embolism.
E. Air embolism.
ANSWER: A
62. All the following statements are true regarding allergic blood transfusion reactions except
A. They are caused by donor antigens or antibodies.
B. They are more commonly occur with transfusion of packed red blood cells.
C. They are usually mild.
D. Usually treated by antihistamines.
E. Epinephrine is occasionally needed in the management of the reaction.
ANSWER: B
63. All the following are true about refeeding syndrome except
A. It occurs more with patients on parenteral nutrition than patients on enteral formulas.
B. It results mainly from decreased levels of some electrolytes.
C. It results in multiorgan failure.
D. It is prevented by increasing calorie intake slowly and regular administration of minerals and vitamins.
E. It is treated by stopping proteins in the given formula.
ANSWER: E
64. In considering a donor for living related liver transplantation, one of the following is a contra indication from
the donor stand point.
A. Replaced or accessory right or left hepatic artery
B. Active Non-Alcoholic Steatohepatitis (NASH) with fibrosis
C. Atherosclerosis of the aorta
D. Two right bile ducts
E. Dual right hepatic vein
ANSWER: B
65. A 43-year-old man was transported to the emergency department after a high voltage electrical injury at
work. After intubation, resuscitation and exposure, you notice a significant burn in his right upper extremity
and his feet. In regards to his electrical burn, all of the following are true except:
A. Admission under cardiac monitoring due to high risk for arrhythmias
B. Electrical burn necrosis may extend beyond the burn entry site in his extremity
C. Repeated debridement and fasciotomy may be necessary
D. Allowing necrotic tissue to spontaneously separate on its own prior to debridement is preferable, as the
extent of necrosis may be extensive
E. Main vessel thrombosis and potential need for amputation is a risk in extremity electrical burns
ANSWER: D
66. While performing a laparoscopic appendectomy in a thin female patient, the consultant surgeon notices a
prominent peristalsing structure crossing the right external iliac artery, traversing to the medial part of the
pelvis. This tube-like structure represents:
A. Right round ligament
B. Right ureter
C. Right broad ligament
D. Right gonadal vessel
E. Hypogastric plexus
ANSWER: B
67. A patient with an open wound inquires about the best option to provide the best aesthetic (cosmetic) result.
Which of the following gives the best cosmetic result?
A. Nothing can be done
B. Applying a vacuum dressing will almost result in a scar-less wound
C. Frequent dressings
D. Placement of silver dressing
E. Delayed primary closure
ANSWER: E
68. What is the most likely initial symptom to occur as a result of local anesthetic toxicity?
A. Seizures
B. Restlessness
C. Slurred speech
D. Coma
E. Muscle weakness
ANSWER: B
69. A 55-year-old man with Crohn’s disease had undergone resection of small bowel and anastomosis. Ten
days later, he is found to have bilious drainage of 1 L/d from the drains. He is started on total parenteral
nutrition (TPN). Four days later, his arterial blood gases (ABGs) are pH, 7.25; PO2, 98 mm Hg; and PCO2,
40 mm Hg. His anion gap is 10. The most likely cause is which of the following?
A. Diabetic ketoacidosis
B. Renal failure
C. Hypovolemic shock
D. Small-bowel fistula
E. Uncompensated metabolic alkalosis
ANSWER: D
70. After undergoing a transurethral resection of the prostate, a 65-year-old man experiences excessive
bleeding attributed to fibrinolysis. It is appropriate to administer which of the following?
A. Heparin
B. Warfarin (Coumadin)
C. Volume expanders and cryoprecipitate
D. Aminocaproic acid (Amicar)
E. Fresh-frozen plasma and vitamin K
ANSWER: D
71. A 70-year-old woman has low cardiac output with increased pulmonary capillary wedge pressure (PCWP)
and increased systemic vascular resistance. What should be the drug of choice?
A. Dopamine
B. Norepinephrine
C. Dobutamine
D. Epinephrine
E. Phenylephrine
ANSWER: C
74. All the following statements about the anatomy of the esophagus are true except
A. The narrowest point of esophagus is at the level of cricopharyngeal muscle
B. The Meissner plexus is located in the submucosa
C. The Auerbach plexus is located between the muscle layers of the esophagus
D. The esophagus lacks the serosal layer
E. The outer longitudinal muscle layer is an extension of the crichopharyngeus muscle
ANSWER: E
75. All the following statements regarding the blood supply of the stomach are true except
A. The left gastroepiploic artery commonly arises from splenic artery
B. Ligation of the left gastric artery can result in acute left sided hepatic ischemia
C. The stomach is well protected from ischemia because of rich collateral circulation
D. The short gastric arteries provide significant blood supply to the body of the stomach
E. The gastroepiploic artery usually originates from the gastroduodenal artery
ANSWER: D
79. All the following are risk factors for surgical site infection except
A. Obesity.
B. Female gender.
C. Uremia.
D. Obesity.
E. Jaundice.
Answer: B
81. Which of the following antibiotics is most effective against Clostridia difficile?
A. Imipenem.
B. Metronidazole.
C. Vancomycin.
D. Ciprofloxacin.
E. Ceftazidime.
ANSWER: C
82. With regard to bipolar and monopolar devices, which of the following is true?
A. Both use direct current (DC).
B. Both uses alternating current (AC).
C. Bipolar uses AC, while monopolar uses DC.
D. Bipolar uses DC, while monopolar uses AC.
E. Neither requires an electrosurgical unit.
ANSWER: B
83. Which of the following sites for central venous catheter placement is associated with the lowest rate of
catheter-associated blood stream infections?
A. Internal jugular vein
B. Subclavian vein
C. Femoral vein
D. Both A and B are equivalent
E. All sites are equivalent if sterile technique is maintained
ANSWER: B
84. All the following are causes of wide anion gap metabolic acidosis except
A. Lactic acidosis.
B. Ketoacidosis.
C. Pancreatic fistula.
D. Alcohol toxicity.
E. Salicylic acid overdose.
ANSWER: C
85. Autologous intraoperative blood donation can be an option in which of the following conditions?
A. Aortocoronary bypass.
B. Mastectomy and breast reconstruction for breast cancer.
C. Sigmoid resection for volvulus.
D. Surgery for abdominal abscess.
E. Liver transplant for cirrhosis and small hepatocellular carcinoma.
ANSWER: A
88. Early after a skin cut wound, all the following occur at the site of injury except
A. Vasodilation.
B. Platelets aggregation.
C. Infiltration by macrophages and polymorphonuclear cells.
D. Release of serotonin.
E. Swelling of the injury site.
ANSWER: A
89. All the following are true regarding necrotizing soft tissue infections except
A. They are usually polymicrobial.
B. Beta hemolytic streptococci are usually part of the causative microbes.
C. Clostridia perfrengens causes gas gangrene.
D. Pain is usually out-of-proportion to physical signs.
E. Treatment is high dose of a wide spectrum antibiotic changed according to culture result when ready.
ANSWER: E
90. A 40-year-old lady presented with 2cm left breast mass in the retro-areolar region proved by a true-cut
biopsy to be invasive ductal carcinoma. Metastatic work up was free. You discussed the condition with her
and offered mastectomy with sentinel axillary LN sampling. She refused to do any breast surgery. What
should you do?
A. Insist on your plan.
B. Discuss the matter with her family.
C. Refer her to another surgeon.
D. Discuss with her other treatment options although not as effective as surgery.
E. Inform the police that she refuses treatment.
ANSWER: D
92. All the following is true about the abdominal wall anatomy except
A. The rectus abdominus muscles lie between the anterior and posterior rectus sheets all the way from
xiphoid to symphysis pubis.
B. The internal oblique muscle lies between the external oblique and the transversus abdominis.
C. The transversalis fascia lies under the transversus abdominis muscle.
D. The latissimus dorsi lies posterior to the external oblique muscle.
E. The linea semilunaris lies at the lateral border of the rectus abdominis.
ANSWER: A
94. All the following about the inguinal anatomy are true except
A. The deep inguinal ring is a defect in the transversalis fascia.
B. The deep inguinal ring lies midway between the symphysis pubis and the anterior superior iliac spine.
C. The inferior epigastric vessels lie just medial to the deep inguinal ring.
D. The external oblique aponeurosis forms the anterior wall of the inguinal canal.
E. The inguinal ligament constitutes the floor of the inguinal canal.
ANSWER: B
96. A 20-year-old male underwent surgery for hemorrhoids, which kind of postoperative analgesia you prefer to
give him in the first 3 days?
A. Paracetamol.
B. Nonsteroidal anti-inflammatory.
C. Local lidocaine gel.
D. Carbamazepine.
E. Diazepam.
ANSWER: B
97. All the following are true about thyroid neoplasms except
A. Follicular neoplasms are unifocal.
B. Papillary cancers carries an excellent prognosis.
C. Papillary cancer metastasize commonly to cervical lymph nodes.
D. Lymph node metastasis in papillary thyroid cancer is the main determinant of prognosis.
E. Medullary cancer commonly metastasizes to lymph nodes.
ANSWER: D
100. All the following statements regarding gastrointestinal tract diverticular disease are true except
A. Most colonic diverticula are false diverticula
B. Meckel's diverticulum is a pulsion type diverticulum
C. The most common way of presentation in children with symptomatic Meckel's diverticulum is
Gastrointestinal bleeding
D. False diverticula are usually acquired
E. Sigmoid colon is the most affected site
ANSWER: B
3rd year:
1. Following a fight, a 21-year-old male has an inability to dorsiflex his right foot (foot drop) while losing
sensation in the dorsum of his foot. The nerve affected is:
A. Medial plantar nerve
B. Lateral plantar nerve
C. Tibial nerve
D. Lateral cutaneous nerve of the calf
E. Common peroneal nerve
ANSWER: E
4. A patient presents with a keloid scar to your clinic. All describe her condition except:
A. It is usually confined to the original scar
B. Can be treated with steroids
C. Can be treated with silicone injections
D. Can be treated with radiation (XRT)
E. Tend to form in upper extremities and sternum more commonly
ANSWER: A
5. The most important prognostic factor for breast cancer devoid of metastases is:
A. Node status
B. Size of the tumor
C. Tumor grade
D. Gender of the patient
E. Age of the patient
ANSWER: A
9. 47-year-old woman presents to the emergency room after a motor vehicle accident, mildly lethargic and
confused, with normal airway and vitals. She complains of a headache, and has a frontal scalp hematoma.
After stabilizing her cervical spine and doing primary survey, she becomes more lethargic, moving only to
pain. Which of the following is the most likely to be present?
A. Subdural hematoma
B. Extradural hematoma
C. Cerebral contusion
D. Intracerebral hematoma
E. Diffuse axonal injury
ANSWER: B
10. A 19-year-old girl sustained a posterior knee dislocation. Her distal pulses are absent. The most
appropriate next step is:
A. Start thrombolytic therapy immediately
B. Start Heparin immediately
C. Exploration of the vessels
D. Angiogram
E. Immediate reduction of the knee
ANSWER: E
11. Following a house fire, a 25-year-old male patient was brought to the emergency room with extensive
partial and full thickness burns. You note black carbon deposits around his nostrils and oropharynx. Which
of the following is an immediate priority?
A. Adequate analgesia
B. Aggressive fluid resuscitation
C. Immediate transfer to a burn unit
D. Secure the airway with higher possible need for intubation
E. Dressing changes to the burn wounds
ANSWER: D
12. There were 192,000 new cases diagnosed with lung cancer in 2019. This means:
A. Incidence
B. Prevalence
C. Mean
D. Mode
E. Median
ANSWER: A
14. All the following are true about hepatic venous drainage except
A. The right hepatic vein drains liver segments V, VI, VII and VIII.
B. The middle hepatic vein drains liver segments IV, V and VIII.
C. The left hepatic vein drains liver segment I.
D. The caudate lobe drains directly into the IVC.
E. The left and middle hepatic veins usually form a common trunk before entering the IVC.
ANSWER: C
15. All the following are true about bile acids and bile salts except
A. Bile salts are responsible for the digestion and absorption of lipids.
B. Bile salts are sodium and potassium salts of bile acids conjugated with amino acids.
C. The bile acids are derivatives of cholesterol.
D. Chenodeoxycholic acid is a secondary bile acid.
E. 90% to 95% of bile salts secreted into the intestine are reabsorbed at the terminal ilium.
ANSWER: D
16. All the following are true about liver hemangiomas except
A. They are the most common solid liver lesions.
B. More common in women.
C. Most are discovered incidentally.
D. Spontaneous rupture is common.
E. Resection can be accomplished mostly by enucleation
ANSWER: D
19. Laparoscopic surgery may induce the following hemodynamic changes except:
A. Increased preload.
B. Increased cardiac output.
C. Increased pulmonary vascular resistance.
D. Increased systemic vascular resistance.
E. Decreased renal perfusion pressure.
Answer: B
22. Following insertion of a Veress needle, what is the initial maneuver to confirm intraperitoneal placement?
A. Saline drop test
B. Aspiration of the needle
C. Flushing the needle
D. Measuring insufflation pressure
E. Starting high-flow insufflation
ANSWER: B
25. Which of the following is true regarding diverticular diseases of the lower GI tract?
A. They occur most commonly in the ascending colon.
B. The rectum can be affected.
C. Asymptomatic sigmoid or cecal diverticuli require surgical management
D. The vast majority of complications from diverticula occur at the sigmoid colon.
E. They are associated with a long, redundant colon.
ANSWER: D
26. A 35 years old male patient developed large amount of fresh bleeding per rectum 10 days post
hemorrhoidectomy. The most probable cause for his bleeding is
A. Sloughing of a vessel wall.
B. Uncontrolled bleeding missed at the time of surgery.
C. Clot dislodgment.
D. Injury by defecation.
E. Vasodilation.
Answer: A
28. All the following are true regarding platelets concentrate except
A. A unit is pooled usually from more than one donor.
B. It is kept on a special agitator.
C. It is stored at 4 degrees centigrade.
D. May transfer bacterial infections.
E. Shelf life is 5 days.
Answer: C
29. All of the following are recognized consequences of inadequate pain relief after upper abdominal surgery
EXCEPT:
A. Decreased functional residual capacity
B. Ileus
C. Deep vein thrombosis
D. Sodium and water retention
E. Reduced systemic vascular resistance
ANSWER: E
30. A 22 year old man in undergoing a rapid sequence induction prior to an appendectomy. The anesthetist
becomes concerned because the patient becomes tachycardia and develops atrial fibrillation. His
temperature is 39.5 oC, blood pressure is slightly elevated. What is the most appropriate course of action?
A. Proceed to immediate laparotomy
B. Administration of broad spectrum intravenous antibiotics
C. Start infusion of inotropes
D. Administration of intravenous Dantrolene
E. Administration of Neostigmine
ANSWER: D
31. Bariatric surgery is used for obesity treatment but how much is the incidence of type 2 diabetes reduced in
obese individuals after surgery?
A. 80-90%
B. No reduction in the incidence
C. Approximately 50 %
D. Approximately 40 %
E. Approximately 30 %
Answer: A
33. Unilateral bloody nipple discharge from one duct orifice is most commonly caused by:
A. Paget's disease of the nipple.
B. Carcinoma in situ.
C. Inflammatory carcinoma.
D. Subareolar mastitis.
E. Intraductal papilloma.
Answer: E
37. A 40-year-old man who weighs 65 kg is being observed in the ICU. Twenty-four hours postoperatively, he
develops convulsions. His serum sodium is 118 mEq/L. Appropriate management includes which of the
following?
A. Administration of normal saline (0.9%)
B. Administration of hypertonic saline (3%)
C. Emergency hemodialysis
D. Administration of vasopressin
E. Administration of Lasix, 40 mg intravenously (IV)
ANSWER: B
38. A 42-year-old man with small-bowel fistula has been receiving TPN with standard hypertonic glucose-
amino acid solution for 3 weeks. The patient is noticed to have scaly, hyperpigmented lesions over the
acral surfaces of elbows and knees, similar to enterohepatic acrodermatitis. What is the most likely cause
of the condition?
A. Copper deficiency
B. Essential fatty acid deficiency
C. Excess glucose calories
D. Hypomagnesemia
E. Zinc deficiency
ANSWER: E
39. Regarding successful day case surgery, one of the following is not true;
A. minimal access techniques.
B. Good hemostasis.
C. Avoidance of unnecessary tissue handling or tension.
D. Provided with adequate analgesia.
E. Major procedures should be scheduled late on list.
ANSWER E
40. Postoperative suppressive treatment with thyroxin has a role in the management of:
A. Follicular adenoma.
B. Papillary carcinoma.
C. Anaplastic carcinoma.
D. Medullary carcinoma.
E. Lymphoma.
Answer: B
41. Bone hunger syndrome can occur after total thyroidectomy for:
A. Papillary thyroid cancer.
B. Multinodular goiter.
C. Medullary thyroid cancer.
D. Graves` disease.
E. Follicular thyroid cancer.
Answer: D
42. The most common site of acute biliary pain (biliary colic) is
A. Subxiphoid.
B. Left hypochondrial.
C. Right flank.
D. Right scapular region.
E. Interscapular.
F. Answer: A
43. All the following are true about amylase in acute pancreatitis except
A. It starts to increase shortly after the onset of symptoms.
B. It peaks within several hours.
C. It remains elevated for 3 to 5 days in average.
D. Its serum levels in general correlate directly with the severity of disease.
E. The laboratory reading of it is commonly normal in acute pancreatitis caused by hyperlipidemia.
Answer: D
44. All of the following are true regarding familial juvenile polyposis EXCEPT ?
A. It is autosomal dominant.
B. The polyps are hamartomatous.
C. Bleeding is common symptom.
D. Once a polyp is detected, total proctocolectomy is recommended.
E. Intussusception is possible presentation.
ANSWER D
45. A 55-year-old male, who is noted to have diarrhea, flushing and bronchoconstriction as well as right-sided
cardiac valvular disease, is most likely to have his primary carcinoid tumor in
A. Appendix
B. Bronchus
C. Rectum
D. Stomach
E. Ileum.
ANSWER E
46. Which of the following is a more common to cause massive colonic bleeding?
A. Cancer.
B. Ulcerative colitis.
C. Angiodysplasia.
D. Diverticulitis.
E. Granulomatous colitis.
ANSWER C
47. A 6 weeks old baby brought by his mother with umbilical discharge since birth. On examination, it was not
smelly and there was what looks like red spot inside the stump. In this case the commonest cause will be:
A. Foreign body in the stump
B. Patent vietello intestinal duct
C. Patent urachus
D. Umbilical granuloma
E. Umbilical sepsis
ANSWER: D
48. A 2 months old baby presented with an umbilical hernia. The treatment is:
A. Observe
B. Herniotomy
C. Apply a truss
D. Inject sclerosing material
E. Truss for one year then operate
ANSWER: A
49. The following best describes the sequence of sensory recovery in a healing skin graft:
A. Temperature, pain, light touch.
B. Pain, temperature, light touch.
C. Pain, light touch, temperature.
D. Light touch, pain, temperature.
E. Temperature, light touch, pain.
ANSWER C
51. In primary hyperparathyroidism which of the following tests is most important in anticipating post-surgery
bone hunger syndrome:
A. Alkaline phosphatase.
B. Erythrocyte sedimentation rate (ESR).
C. C reactive protein.
D. Preoperative blood calcium level.
E. Preoperative blood phosphate level.
ANSWER: A
53. Which of the following is more common to cause massive colonic bleeding?
A. Cancer.
B. Ulcerative colitis.
C. Angiodysplasia.
D. Diverticulitis.
E. Granulomatous colitis.
ANSWER: C
54. Which pelvic floor muscle provides an important function in maintaining the rectal angle?
A. Pubococcygeus.
B. Iliococcygeus.
C. Puboiliacus.
D. Puborectalis.
E. External and internal sphincter.
ANSWER: D
56. The treatment of choice for type III choledochal cyst (choledochocele) is
A. Excision.
B. Sphincterotomy.
C. Whipple operation.
D. Observation.
E. Biliary bypass.
ANSWER: B
58. A 24-year-old man was involved in a motor vehicle accident. His car caught fire with evidence of
carbonaceous sputum, edematous oropharynx, tachypnea, and stridor. The patient is hemodynamically
stable with oxygen saturation of 97%. Before attempting intubation, which of the following maneuvers is
necessary?
A. Insert an endotracheal tube for his inhalational injury
B. Axial stabilization of his cervical spine
C. Administer paralyzing agents
D. Administer morphine for sedation
E. Immediate needle cricothyroidotomy
ANSWER: B
59. During cholecystectomy in a 67-year-old woman, there is severe bleeding from accidental injury to the
hepatic artery. The patient requires transfusion of 2000 ML of blood. After the operation, 24-hour urine
output varies between 1250 and 2700 mL/d. She was adequately hydrated, but BUN levels continue to rise
10–12 mg daily over a 5-day period. What is the most p0ssible explanation?
A. Progressive bleeding
B. High-output renal failure
C. Post cholecystectomy syndrome
D. Glomerulonephritis
E. Obstructive jaundice
ANSWER: B
61. Axillary lymph node dissection is not an option in treating one of the following conditions:
A. 2cm pure non comedo type intraductal carcinoma.
B. 1cm infiltrating lobular carcinoma.
C. 8mm infiltrating ductal carcinoma.
D. A pure medullary cancer in the upper inner quadrant.
E. 3cm non otherwise specified invasive ductal carcinoma.
ANSWER: A
62. In below knee amputation for a diabetic foot; Tibia should be transected:
A. 4-8 cm below tibial plate.
B. 8- 12 cm below tibial plate.
C. 12-16 cm below tibial plate.
D. 16- 20 cm below tibial plate.
E. 20- 24 cm below tibial plate.
ANSWER: B
63. Indications for operation in a patient with primary hyperparathyroidism include all of the followings except:
A. Age older than 50.
B. Nephrolithiasis.
C. A substantial decline in renal function.
D. A substantial decline in bone mass.
E. Depression and fatigue.
ANSWER: A
64. The most important and proved benefit of external beam radiation after wide excision of an invasive breast
cancer is the ability to improve one of the following outcomes:
A. Recurrence of cancer in the ipsilateral breast.
B. Patient survival time.
C. Regional nodal recurrence.
D. Patient cure rate.
E. Distant metastasis upon follow up.
ANSWER: A
65. Which of the following is true regarding the management of short bowel syndrome?
A. Glutamine should be avoided.
B. Octreotide is the cornerstone of management.
C. Proton pump inhibitor is contraindicated.
D. Early enteral feeding is indicated.
E. Patients who still require TPN after 3 months will require permanent TPN.
ANSWER D
66. Which of the following pancreatic tumors has the lowest risk of malignancy?
A. Mucinous cystic neoplasm.
B. Serous cystic neoplasm.
C. Intraductal papillary mucinous neoplasm.
D. Solid pseudopapillary tumor.
E. Acinar cell tumor.
ANSWER: B
67. With regards to genetic susceptibility to cancer, one of the following is true:
A. All women with BRCA 1 mutation develop breast cancer
B. RET-protoncogene is uniquely associated with colon cancer
C. P53 is rarely found in breast cancer
D. BRCA2 may be associated with ovarian cancer
E. MSH2 & MLH2 are present in familial adenomatous polyposis coli
ANSWER: D
68. In regard to the following physical findings, one of the following is true:
A. Virchow’s node is an early sign of abdominal malignancy
B. Sister Mary Joseph’s sign is a periumbilical tumor deposits from intra-abdominal malignancy
C. Blumer’s shelf is due to a direct extension of a rectal tumor
D. Horner’s syndrome is due to invasion of the thoracic sympathetic chain
E. Krukenberg’s tumor is a primary ovarian malignancy
ANSWER: B
69. A 40- year-old 80 -Kg man sustained a 40% burn of the total surface area, with a significant inhalational
injury. After intubation and initial resuscitation and admission, the parkland formula was calculated and
started lactated ringer’s solution at 800cc/hr. Six hours later, the patient became oliguric. What is the best
management to follow?
A. Increase the volume of lactated ringer’s solution infused
B. Bolus of Hetastarch solution
C. Trial of small dose of furosemide
D. Low-dose dopamine (vasopressor) infusion
E. Insert Swanz-Ganz catheter and measure pulmonary wedge pressure
ANSWER: A
70. A male infant, born at term, appears well following delivery. Six hours later, he is noted to have bilious
vomiting by the pediatricians. On examination, he seems well and his abdomen is soft and non-tender.
What is the best course of action?
A. Arrange an abdominal x-ray
B. Undertake a test feed
C. Perform serial abdominal examinations
D. Arrange an upper GI contrast study
E. Arrange a laparotomy
ANSWER: D
72. You have just completed a laparotomy for peritonitis due to a perforated peptic ulcer. What is the best
surgical strategy for avoidance of a complete abdominal wound dehisence?
A. Use of skin clips to close the skin rather than sub cuticular sutures
B. Careful approximation of the peritonum with non absorbable sutures
C. Mass closure of the midline wound using a 1 polydiaxone suture
D. Direct apposition of the rectus muscle rather than linea alba aponeurosis
E. Mass closure of the midline wound using a 3/0 polypropylene suture
ANSWER: C
73. A 24-year-old man who is admitted to the intensive care unit (ICU) following severe head injury develops
seizures on the fourth day of hospitalization. His urine output is 500 mL over 24 hours, sodium is 115
mEq/L, and serum and urine osmolality are 250 and 800 mOsm, respectively. The metabolic abnormality is
due to which of the following?
A. Administration of D5W (5% dextrose in water) and 0.33 normal saline
B. Syndrome of inappropriate secretion of antidiuretic hormone (SIADH)
C. Decreased antidiuretic hormone (ADH) secretion
D. Nasogastric suction
E. Renal insufficiency
ANSWER: B
74. A 22 years old male patient, presented with sever dyspnea and chest discomfort for 30 minutes, with
distended neck veins and hypotension, which of the following should be done first?
A. Insertion of a large bore needle in the chest.
B. Chest x-ray.
C. ECG.
D. Insert a chest tube.
E. Endotracheal intubation.
ANSWER: A
75. All of the following are true regarding fresh frozen plasma (FFP) except
A. Stored at -40 degrease C.
B. Shelf life is 6 months.
C. Rh D positive FFP can be given to Rh D negative patients.
D. It is a source for vitamin dependent clotting factors.
E. It is a source of factor V.
ANSWER: B
77. Necrotizing soft tissue infections are associated with all of the following except
A. Pain.
B. Skin blistering.
C. Skin involved more than the subcutaneous tissue.
D. Greyish discharge.
E. Focal skin gangrene in some cases.
ANSWER: C
78. All of the following about nylon sutures are true except
A. It is a poly amide polymer.
B. It loses 15% to 20% of its tensile strength per year.
C. It never degrades.
D. Has low tissue reaction.
E. Can be used for skin closure.
ANSWER: C
79. All of the following are true regarding surgical audit except
A. It aims to improve patients’ care.
B. It involves comparing aspects of care against defined standards.
C. It can be an essential component for revalidation of the surgeon.
D. If the results are far from standards the surgeon should stop working.
E. An audit should answer predetermined questions.
ANSWER: D
80. Regarding rapid sequence induction of anesthesia, all of the following are true except
A. Used to allow the airway to be rapidly secured.
B. Used for patients with high risk of regurgitation.
C. Intravenous anesthetic agents can be used.
D. Slow acting muscle relaxant is used.
E. Commonly needed in emergency surgeries.
ANSWER: D
81. All of the following have less favorable prognosis in skin squamous cell carcinoma except
A. Depth more than 6mm.
B. High histological grade.
C. Surface size more than 2 cm.
D. Tumors on the trunk.
E. Tumors that arise in burn scars.
ANSWER: D
82. All the following statements concerning caustic esophageal injury with an alkaline substance (lye) are true
except
A. Injury to esophagus is the result of liquefactive necrosis
B. Endoscopy should not be performed within the first 72 hours because of the risk of perforation
C. Lye can be neutralized with half strength vinegar if the patient is seen in the first hour of ingestion
D. To decrease long term stricture rate, endoscopic dilatation should be performed only after re-
epithelialization
E. As compared to injury caused by of acidic liquid, alkaline substances cause more deep tissue penetration
ANSWER: B
83. When comparing the intestinal subtype to the diffuse subtype of gastric adenocarcinoma, which of the
followings is true
A. Intestinal subtype is more common in younger patients.
B. intestinal subtype carries worse prognosis.
C. intestinal subtype usually has no identifiable precursor.
D. intestinal subtype usually arises in the cardia.
E. intestinal subtype usually forms discrete ulcer or mass.
ANSWER: E
84. All the following statements regarding Gastrointestinal Stromal tumors (GISTs)are true except
A. The diagnosis of GIST is based on the presence of more than 10 mitotic figures on high-power field
microscopy.
B. GISTs usually don't metastasize to lymph nodes.
C. GISTs are usually resistant to conventional chemotherapy and radiotherapy.
D. Extra gastric GISTs carry worse prognosis.
E. Complete surgical resection is the standard of treatment.
ANSWER: A
86. All the following are true about coronary arteries (CA) anatomy except
A. They are branches of the ascending aorta.
B. They arise from ostia in the aortic sinuses.
C. Three coronary arteries arise from the aorta.
D. The worst CA stenosis prognosis is that located at the left main CA.
E. The right CA passes anteriorly between the right atrial appendage and the pulmonary trunk.
ANSWER: C
87. All the following are true about the venous ulcer except:
A. Usually has a sloping edge.
B. The floor contains granulation tissue covered by slough and exudate.
C. An elevation in the edge may indicate malignant change.
D. Characteristically develop at the gaiter’s area.
E. Most of them develop at the lateral aspect of the calf.
ANSWER: E
88. All the following are true about anal tumors except:
A. The commonest serotype of human papilloma virus that causes anal squamous cell carcinoma (SCC) is
type 6 serotype.
B. Bleeding is the commonest presentation for SCC.
C. The first line treatment for SCC is chemoradiation.
D. Anal adenocarcinoma is more aggressive than SCC.
E. Symptomatic anal GIST carries worse prognosis.
ANSWER: A
89. Regarding Warthin tumor of the salivary glands all are true except
A. There is no risk of malignancy.
B. Can be treated by enucleation.
C. It may be cystic.
D. Occurs in male in 90% 0f cases.
E. Occurs equally in parotid and submandibular glands.
ANSWER: E
90. The first branch to the colon from the superior mesenteric artery is
A. Ileocolic.
B. Right colic.
C. Middle colic.
D. Left colic.
E. Superior rectal.
ANSWER: C
91. A 6-year-old male presented to the emergency room with his mother, his mother said that he felt from the
second floor. He was conscious, talking and walking, his pulse was 150 per minute, his pressure 90 over
50 mmHg. He has bruises over the scalp and chest. Within the last 6 months he was presented to the
emergency twice for falling down, one of them he had leg fracture. All the following are suitable actions to
do with this child except
A. Admit him to the ICU.
B. Take blood sample for CBC.
C. Do FAST ultrasound.
D. Do chest X-ray.
E. Inform child protection authorities about the case.
ANSWER: A
92. A 35-year-old female who is known to have insulin dependent diabetes, operated for perforated duodenal
ulcer. 6 hours after surgery you were called to see her, she was stuporous, has tachycardia, normal pulse
with urine output 1200cc over 6 hours. You asked for lab tests and revealed Hb 14.5, WBC 15000, blood
sugar 550 mg per dL. She most likely has
A. Hypernatremia.
B. Post-operative bleeding.
C. Wide anion gap metabolic acidosis.
D. High output renal failure.
E. Overhydration.
ANSWER: C
93. A 68-year-old smoker male presented with 6 months history of pain during walking in his right thigh and
calf. You suspect that he has a significant disease at which of the following arteries?
A. Aorta.
B. Common iliac.
C. Femoral bifurcation.
D. Superficial femoral.
E. Deep femoral.
ANSWER: B
96. All the following are true about liver hydatid cysts except
A. Echinococcus granulosus is the main causative organism.
B. The adult worm present in the dog intestine.
C. The causative organism is a tapeworm.
D. In human, the ova pass to the liver through intestinal lymphatics.
E. Ultrasound is a good screening test for the condition.
ANSWER: D
97. After cholecystectomy you opened the gall bladder and found -in addition to stones- pink to red mucosa
with yellow elevations looking like strawberry. This appearance in called
A. Cholesterosis.
B. Cholesterol polyposis.
C. Cholecystitis glandularis proliferans.
D. Intramuscular stones.
E. Typhoid cholecystitis.
ANSWER: A
98. A barium swallow is good in identifying all the following pathologies except
A. Large hiatal hernia.
B. Gastroesophageal reflux.
C. Esophageal motility disorders.
D. Esophageal strictures.
E. Barrett’s mucosa
ANSWER: E
99. All the following are alarming symptoms that indicate the need for upper GI endoscopy in a 40-year-old
female except
A. Recurrent epigastric pain.
B. Weight loss.
C. Dysphagia.
D. Frequent recurrent vomiting.
E. Anemia.
ANSWER: A
100. All the following about adenomatosis polyposis coli APC gene are true except
A. It is a tumor suppressor gene.
B. APC defects are present in patients with familial adenomatosis polyposis FAP.
C. APC defects are rarely present in sporadic cases of colorectal cancer.
D. Mutations of both alleles of APC gene is necessary to initiate polyp formation.
E. In FAP the site of mutation in the APC gene correlates with the disease severity.
ANSWER: C
105. Which of the following inhalation anesthetics accumulates in air-filled cavities during general anesthesia?
A. Diethyl ether
B. Nitrous oxide
C. Halothane
D. Methoxyflurane
E. Trichloroethylene
ANSWER: B
106. Which of the following fractures or dislocations of the extremities induced by blunt trauma is associated
with significant vascular injuries?
A. Knee dislocation
B. Closed posterior elbow dislocation
C. Midclavicular fracture
D. Supracondylar femur fracture
E. Tibial plateau fracture
ANSWER: A
107. All of the following arises from the descending aorta except:
A. Nine pairs of posterior intercostal arteries.
B. Four-Five esophageal branches
C. Two to three bronchial arteries.
D. Left vertebral artery.
E. Mediastinal branches.
ANSWER: D
112. Which of the following is the most common symptom of nasopharyngeal cancer?
A. Nasal bleeding
B. Cervical mass
C. Dry eyes
D. Ophthalmoplegia
E. Diplopia
ANSWER: B
113. These are contents of the posterior triangle of the neck except:
A. Occipital artery.
B. Transverse cervical artery.
C. External jugular vein.
D. Hypoglossal nerve.
E. Brachial plexus.
ANSWER: D
116. The spinal accessory nerve passes through the posterior cervical triangle:
A. Superficial to the platysma muscle.
B. In the superficial cervical fascia.
C. Between the investing fascia and the prevertebral fascia.
D. Within the carotid sheath.
E. Deep to the prevertebral fascia.
ANSWER: B
121. All the muscles of the larynx are innervated by recurrent laryngeal nerve except:
A. Cricothyroid.
B. Posterior cricoarytenoid
C. Lateral cricoarytenoid.
D. Interarytenoid.
E. Thyroarytenoid.
ANSWER: A
4th year:
1. The peritoneal space located to the left and posterior to Foramen of Winslow is:
A. Rutherford Morison’s pouch (hepatorenal pouch)
B. Right subphrenic space
C. Right paracolic gutter
D. Lesser sac
E. Left spleno-renal space
ANSWER: D
3. A 60-year-old man with unresectable lung cancer is undergoing radiation therapy (XRT). The main target of
XRT for malignancy treatment is:
A. DNA
B. RNA
C. Cell wall
D. Proteins
E. Cytoplasm
ANSWER: A
4. A 41-year-old large-surface-burn patient, is intubated in the ICU. After 10 hours of aggressive hydration,
escharotomy and fasciotomy, her urine output is becoming scanty. After placing a central line, her CVP is
18. The most likely cause of her renal failure is:
A. Inadequate fluid resuscitation
B. Cardiogenic shock
C. Sepsis
D. Pulmonary embolus
E. Myoglobinuria
ANSWER: E
5. One of the following injuries is associated with permanent damage to recurrent laryngeal nerve during
thyroid surgery:
A. Neuropraxia.
B. Axonotemesis.
C. Neurotemeisis.
D. Dendridotemisis.
E. Thermal injury
ANSWER: C
6. For a patient with a thyroid nodule, the following ultrasonic feature has the lowest potential for malignancy:
A. Hypo-echogenicity.
B. Micro calcifications.
C. Spongiform appearance.
D. 40% of its size is a cyst.
E. 2 cm in diameter.
ANSWER: C
7. All of the followings are true regarding neuroendocrine tumors of the pancreas except:
A. Insulinomas are the commonest functioning form.
B. Diarrhea is a common presentation in gastrinomas.
C. Peptic ulcers in gastrinomas are usually small and solitary.
D. VIPomas present with diarrhea.
E. They occur in 40–60 per cent of MEN-1 patients.
ANSWER: C
9. All the following are true regarding magnetic resonance imaging (MRI) for evaluation of breast abnormalities
except
A. It is useful for finding the primary breast lesion in patients with positive axillary nodes and negative breast
examination and mammogram.
B. It is more accurate than mammography in diagnosing invasive lobular cancer.
C. It is more helpful than mammography in assessing tumor extent particularly in older women.
D. Its sensitivity in detecting invasive cancer is greater than 90%.
E. Its use as a screening tool is not settled.
ANSWER: C
11. All the following are true about gallbladder cancer except
A. It is more common in females.
B. It carries bad prognosis.
C. Most are associated with gallbladder stones.
D. Most are squamous cell carcinomas.
E. Porcelain gallbladder carries relatively high incidence of the disease.
ANSWER: D
14. All the following are complications associated with pneumoperitoneum except
A. Hyperthermia
B. Acidosis
C. Cardiac arrhythmias
D. Gas embolism
E. Reduced cardiac return
Answer: A
15. During laparoscopic surgery, pneumoperitoneum usually results in a fall in cardiac output when intra-
abdominal pressure exceeds
A. 10 mmHg
B. 20 mmHg
C. 30 mmHg
D. 40 mmHg
E. 50 mmHg
Answer: B
16. Which of the following therapies is proven to reduce mortality and morbidity in bleeding peptic ulcers?
A. Endoscopic procedures
B. H2 antagonists
C. Proton pump inhibitors
D. Octreotide
E. Antacids
Answer: A
19. A 3-year-old girl presents to the ED with crampy abdominal pain for 24 hours. The pain is increasing in
frequency and duration. Ultrasound demonstrates a target sign. The most common pathologic lead point is:
A. Appendix
B. Small bowel polyp
C. Ectopic pancreatic tissue
D. Meckel diverticulum
E. Intraluminal hematoma
ANSWER: D
20. Electromagnetic interference is a concern for patients with pacemakers when using energy devices. Of the
following, which device would not generate any electromagnetic interference?
A. Monopolar electrosurgical device.
B. Monopolar endoscopic device
C. Radiofrequency ablation (RFA)
D. Ultrasonic device
E. Bipolar electrosurgical device
ANSWER: D
21. A 32-year-old female patient is about to undergo an elective repair of a paraumbilical hernia. Which of the
following skin preparation agents would be most appropriate to use?
A. 2% aqueous chlorhexidine
B. B. 2% alcoholic chlorhexidine
C. C. 0.5% alcoholic chlorhexidine
D. Aqueous povidone-iodine solution
E. Isopropyl alcohol
ANSWER: B
22. Common causes of colorectal anastomotic breakdown include all of the following except:
A. Poor blood supply to the bowel edges
B. Short rectal stump
C. Emergency operation
D. Hand-sewn anastomosis
E. Poor technique
ANSWER D
23. An 85-year-old male patient with a history of chronic constipation presents with acute severe colicky
abdominal pain and absolute constipation. Plain abdominal film shows a grossly dilated oval of large bowel
arising from the left lower quadrant. A diagnosis of sigmoid volvulus is made. The next step in management
is:
A. Laparotomy
B. Sigmoidoscopy with flatus tube insertion
C. Sigmoid colectomy with colostomy
D. Barium swallow
E. Computed tomography
ANSWER B
24. A 22-year-old man in undergoing a rapid sequence induction prior to an appendectomy. The anesthetist
becomes concerned because the patient becomes tachycardia and develops atrial fibrillation. His
temperature is 39.5 C, blood pressure is slightly elevated. What is the most appropriate course of action?
A. Proceed to immediate laparotomy
B. Administration of broad spectrum intravenous antibiotics
C. Start infusion of inotropes
D. Administration of intravenous Dantrolene
E. Administration of Neostigmine
ANSWER: D
26. Unilateral bloody nipple discharge from one duct orifice is most commonly caused by:
A. Paget's disease of the nipple.
B. Carcinoma in situ.
C. Inflammatory carcinoma.
D. Subareolar mastitis.
E. Intraductal papilloma.
Answer: E
27. A 49-year-old lady with breast cancer is identified as having a 1mm foci of malignant cells in a sentinel
node. What is the term best used to describe this finding?
A. Isolated tumor cells
B. N1 disease
C. Micro metastasis
D. N2 disease
E. n3 disease
ANSWER: C
28. A 40-year-old man who weighs 65 kg is being observed in ICU. 24 hours postoperatively, he develops
convulsions. His serum sodium is 118 mEq/L. Appropriate management includes which of following?
A. Administration of normal saline (0.9%)
B. Administration of hypertonic saline (3%)
C. Emergency hemodialysis
D. Administration of vasopressin
E. Administration of Lasix, 40 mg intravenously (IV)
ANSWER: B
29. A 42-year-old man with small-bowel fistula has been receiving TPN with standard hypertonic glucose-
amino acid solution for 3 weeks. The patient is noticed to have scaly, hyperpigmented lesions over the
acral surfaces of elbows and knees, similar to enterohepatic acrodermatitis. What is the most likely cause
of the condition?
A. Copper deficiency
B. Essential fatty acid deficiency
C. Excess glucose calories
D. Hypomagnesemia
E. Zinc deficiency
ANSWER: E
31. Bone hunger syndrome can occur after total thyroidectomy for:
A. Papillary thyroid cancer.
B. Multinodular goiter.
C. Medullary thyroid cancer.
D. Graves` disease.
E. Follicular thyroid cancer.
Answer: D
32. Multiple Endocrine Neoplasia type 2 (MEN 2) syndrome is associated with a germ line mutations in:
A. The p53 tumor suppressor gene.
B. The H-ras gene.
C. The N-myc gene.
D. The RET proto-oncogene.
E. The BRAF proto-oncogene.
Answer: D
33. In primary hyperparathyroidism; the following test has an importance in anticipating post-surgery bone
hunger syndrome:
A. Alkaline phosphatase.
B. Erythrocyte sedimentation rate (ESR).
C. C reactive protein.
D. Preoperative blood calcium level.
E. Preoperative blood phosphate level.
Answer: A
34. All the following are true about amylase in acute pancreatitis except
A. It starts to increase shortly after the onset of symptoms.
B. It peaks within several hours.
C. It remains elevated for 3 to 5 days in average.
D. Its serum levels in general correlate directly with the severity of disease.
E. The laboratory reading of it is commonly normal in acute pancreatitis caused by hyperlipidemia.
Answer: D
35. 57A 45 years old female who has liver cirrhosis, she has mild ascites, no encephalopathy but she has
grade 2 esophageal varices. Her hemoglobin is 10 gm/dl, platelets count 70000 per cubic mm, INR 2,
serum creatinine 2mg/dl, serum albumin 3 g/dl, and serum bilirubin 4 mg/dl. What is his Child-Turcotte-
Pugh score?
A. 6.
B. 8.
C. 10.
D. 12.
E. 14.
Answer: C
36. All of the following are true regarding familial juvenile polyposis except
A. It is autosomal dominant.
B. The polyps are hamartomatous.
C. Bleeding is common symptom.
D. Once a polyp is detected, total proctocolectomy is recommended.
E. Intussusception is possible presentation.
ANSWER D
37. In which portion of the gastrointestinal tract are proteins mostly absorbed?
A. Stomach.
B. Duodenum.
C. Jejunum.
D. Ileum.
E. Colon.
ANSWER: C
38. In neoplasia, cells progress from normal to cancerous. One of the following is true:
A. Hyperplasia in the absence of atypia or dysplasia, has minimal or no risk of cancer
B. Atypia changes are irreversible
C. Metaplasia describes cells that have altered size, shape & organization
D. All dysplastic tissue progress to carcinoma
E. Hamartoma carries the highest risk of progression to cancer
Answer: A
39. A 2 months old baby presented with an umbilical hernia. The treatment is:
A. Observe
B. Herniotomy
C. Apply a truss
D. Inject sclerosing material
E. Truss for one year then operate
ANSWER: A
40. A 6-day-old baby was born prematurely at 33 weeks. He has been suffering from respiratory distress
syndrome and has been receiving ventilatory support on Neonatal ICU. He has developed abdominal
distension and is increasingly septic. Ultrasound of the abdomen shows free fluid and evidence of small
bowel dilatation. His blood pressure has remained labile despite inotropic support. What is the best course
of action?
A. Laparoscopy
B. Laparotomy
C. Contrast enema
D. Upper GI contrast study
E. MRI abdomen
ANSWER: B
41. The following best describes the sequence of sensory recovery in a healing skin graft:
A. Temperature, pain, light touch.
B. Pain, temperature, light touch.
C. Pain, light touch, temperature.
D. Light touch, pain, temperature.
E. Temperature, light touch, pain.
ANSWER C
44. A 32-year-old woman is brought to the emergency room with obvious head and extremity injuries. Primary
survey shows she is apneic. Which method is a definitive airway that is best provided immediately?
A. Orotracheal intubation
B. Nasotracheal intubation
C. Cricothyroidotomy
D. Tracheostomy
E. Laryngeal mask airway
Answer: A
45. A 23-year-old man presents to the emergency room with a stab wound to his anterior border of the
sternocleidomastoid muscle, 1 cm above the cricoid cartilage with penetration to the platysma. His vitals
and oxygen saturation are normal, with no other physical findings. Which of the following is considered best
management?
A. Discharge the patient home, as everything seems normal
B. Observe for 6 hours, if patient drinks well, with no respiratory distress then discharge
C. Observe without testing for 24 hours. If no further complaints, then discharge
D. Perform a CT-angiogram. If no carotid injury, then can be discharged
E. Formal neck exploration or perform upper endoscopy with esophagogram and angiogram
Answer: E
46. You prescribed silver sulfadiazine ointment (e.g. Flamazine) to a patient who sustained a third-degree burn
to his dorsal right upper extremity. One of the following is a potential side effect that can occur from this
ointment:
A. Hyponatremia
B. Hypocalcemia
C. Methemoglobinemia
D. Hyperchloremia acidosis
E. Neutropenia
Answer: E
47. A 24-year-old man was involved in a motor vehicle accident. His car caught fire with evidence of
carbonaceous sputum, edematous oropharynx, tachypnea, and stridor. The patient is hemodynamically
stable with oxygen saturation of 97%. Before attempting intubation, which of the following maneuvers is
necessary?
A. Avoid oxygenation with his inhalational injury
B. Axial stabilization of his cervical spine
C. Administer paralyzing agents
D. Administer morphine for sedation
E. Immediate needle cricothyroidotomy
Answer: B
49. When vein grafts are used to bridge intra-arterial defects: one of the following is true?
A. The vein wall thins due to the increased luminal pressure.
B. Graft length decreases by 25-30% long term.
C. The graft stretches by 20-30%
D. The vein wall thickens significantly.
E. There is no ingrowth of smooth muscle cells.
ANSWER D
50. All the following are true about arterial aneurysm except
A. The definition is increase in the vessel diameter more than 50%
B. False aneurysm has a wall consist of a single fibrous layer.
C. Mycotic aneurysm caused by fungal infection.
D. False aneurysm is commonly a sequelly of trauma.
E. Abdominal aortic aneurysm is the most common type of large vessel aneurysm.
ANSWER: C
53. Which of the following arteries is most likely to be involved with serious atherosclerosis?
A. The right coronary artery.
B. The left coronary artery.
C. The left anterior descending coronary artery.
D. The circumflex coronary artery.
E. The obtuse marginal artery.
ANSWER: C
56. Biliary radionuclide scanning as HIDA scan can be valuable in diagnosis of the following conditions except:
A. Hepatocellular dysfunction.
B. Biliary atresia.
C. Common bile duct injury.
D. Acute cholecystitis.
E. Bile leak.
ANSWER: A
57. Ghrelin
A. Decreases insulin release.
B. Decreases glucagon release.
C. Inhibits GI secretion.
D. Inhibits pancreatic exocrine secretion.
E. Is released from pancreatic beta cells.
ANSWER: A
58. After significant blunt trauma, a patient arrives to the emergency room with Glasgow Coma Scale (GCS) of
4/15. His oxygen saturation is poor at 89% after applying a non-rebreathing mask. His eyes are bruised
and blood-stained fluid is coming out of his ears. The method of airway management that is best avoided:
A. Oropharyngeal intubation
B. Laryngeal mask airway
C. Cricothyroidotomy is oropharyngeal is unsuccessful
D. Nasopharyngeal intubation
E. Oropharyngeal intubation with bronchoscopy guidance
ANSWER: D
59. A 48-year-old man with circumferential burn in his right arm, presented to the emergency room. After initial
assessment and resuscitation, he started complaining of increasing pain, paresthesia and weakening
peripheral pulses. Which is the best next action?
A. Angiography
B. Escharotomy
C. Electrolyte assay and replacement
D. Fluid resuscitation
E. Fasciotomy
ANSWER: B
60. If a patient undergoing thoracotomy receives intercostal blocks with bupivacaine, his or her postoperative
period will
A. Be little different from controls
B. Show marked improvement in respiratory function over controls
C. Show little difference in vital capacity but marked pain relief
D. Be marked by hyperventilation
E. Be marked by increased incidence of atelectasis
ANSWER: B
61. All of the following may be acceptable operative approaches to management of the thoracic outlet
syndrome except:
A. Scalenectomy.
B. Excision of a cervical rib.
C. Thoracoplasty.
D. First rib resection.
E. Division of anomalous fibromuscular bands.
ANSWER: C
62. Lateral neck dissection for papillary thyroid tumor includes lymph nodes in:
A. Levels 1, 2 and 3.
B. Levels 2, 3 and 4.
C. Levels 3, 4 and 5.
D. Levels 4, 5 and 6.
E. All levels from 1 to 6.
ANSWER: B
65. One of the following is true regarding short bowel syndrome in adults?
A. Competent ileocecal valve is associated with decreased malabsorption.
B. It is defined as less than 300 cm of residual small bowel.
C. Resection of the ileum is better tolerated than resection of the jejunum.
D. The presence of an intact colon does not alter the severity.
E. It is most commonly caused by multiple operations requiring small bowel resection.
ANSWER: A
66. A 34 years old a symptomatic female has an incidental finding of right liver lobe mass in ultrasound. CT
scan showed a well circumscribed 5 by 5 cm mass with homogenous enhancement in the arterial phase
with central scar and isodense in venous phase. The most probable diagnosis is
A. Hepatic adenoma.
B. Hemangioma.
C. Focal nodular hyperplasia.
D. Hepatocellular carcinoma.
E. Cholangiocarcinoma.
Answer: C
67. During a gastrectomy for cancer, you attending surgeon asks you what lymph nodes are not involved D2
lymphadenectomy:
A. Hepatic artery lymph nodes
B. Splenic artery lymph nodes
C. Celiac trunk lymph nodes
D. Left portal vein lymph nodes
E. Para-aortic lymph nodes
ANSWER: E
68. Which of the following would be requires a biopsy confirming the presence of cancer first prior to definitive
management?
A. An ‘apple core’ lesion in the ascending colon
B. A pancreatic head mass with painless jaundice
C. A palpable breast mass with skin puckering
D. Highly vascular painful mass in the thigh, abutting but not invading the neurovascular bundle
E. A 2 cm parotid mass with an intact facial nerve
ANSWER: C
69. On exploration of a 43-year-old male patient who sustained blunt trauma with pelvic fracture and free
abdominal fluid on FAST ultrasonography, you find a central (Zone I) expanding hematoma and a
contained pelvic (Zone III) hematoma. Which of the following is the best method of management?
A. Observation of both hematomas
B. Explore both hematomas
C. Observe the central hematoma, but explore the pelvic hematoma
D. Just apply an external pelvic fixator and close the abdomen since no free blood is present
E. Explore the central hematoma after obtaining proximal and distal control, and observe the pelvic hematoma
ANSWER: E
70. A 14 month old child is admitted with colicky abdominal pain and on investigation is found to have an ileo-
ileal intussusception. What is the best course of action?
A. Attempt hydrostatic reduction with barium enema
B. Attempt pneumatic reduction with air insufflation
C. Undertake a laparotomy
D. Undertake a colonoscopy
E. Undertake a flexible sigmoidoscopy
ANSWER: C
71. A patient is being weaned from mechanical ventilation. Weaning parameters are obtained prior to deciding
on extubation. Successful weaning from a ventilator is suggested by the presence of which of the
following?
A. An alveolar arterial gradient of more than 350 mm Hg
B. APaO2/FiO2 ratio of <200
C. APaCO2 over 55 mm Hg
D. A tidal volume of over 5 mL/kg
E. A minute ventilation of 12 L/min
ANSWER: D
73. A 25 years old female who underwent sleeve gastrectomy for obesity 6 months ago, presented with 3
months history of vomiting, then she started visual disturbance and inability to stand, then rapidly
developed deterioration in the level of consciousness. Mostly she has
A. Sever protein malnutrition.
B. Hypoglycemia.
C. Dumbing syndrome.
D. Thiamin deficiency.
E. Steatohepatitis.
ANSWER: D
74. All of the following conditions are associated with low cardiac output except
A. Early septic shock.
B. Cardiac tamponade.
C. Tension pneumothorax.
D. Massive pulmonary embolism.
E. Air embolism.
ANSWER: A
75. Capacitance coupling may injure organs in laparoscopic surgery. What is best way to eliminate this risk?
A. Decrease the coagulation power.
B. Use entirely plastic ports.
C. Put pads around hollow organs.
D. Immerse the tissues in normal saline.
E. Use 30 degrees lens.
ANSWER: B
77. All of the following are true regarding amebic infection except
A. The vast majority of carriers are asymptomatic.
B. Amebic liver abscess needs drainage usually.
C. The parasite reaches the liver via the portal circulation.
D. Most of amebic liver abscesses located in the right liver lobe.
E. Ameboma presents as a colonic mass.
ANSWER: B
78. All of the following carry a higher risk of post ERCP pancreatitis except
A. Male gender.
B. Young age.
C. Normal serum bilirubin.
D. Balloon dilatation of the papillary sphincter.
E. Previous ERCP-related pancreatitis.
ANSWER: A
79. All the following statements about the esophageal sphincters are true except
A. The upper and lower esophageal sphincters are high pressure zones rather than actual anatomic
landmarks.
B. On swallowing, the upper esophageal sphincter pressure reaches up to 90 mmHg.
C. The inferior constrictor muscle is the main contributor to the upper esophageal sphincter high pressure
zone.
D. Vagal mediated relaxation of the lower esophageal sphincter occurs during food transit.
E. Gastrin and motilin increase lower esophageal pressure.
ANSWER: C
80. When comparing the intestinal subtype to the diffuse subtype of gastric adenocarcinoma, which of the
followings is true
A. Intestinal subtype is more common in younger patients.
B. Intestinal subtype carries worse prognosis.
C. Intestinal subtype usually has no identifiable precursor.
D. Intestinal subtype usually arises in the cardia.
E. Intestinal subtype usually forms discrete ulcer or mass.
ANSWER: E
81. A 50-year-old postmenopausal woman with no family history of breast cancer underwent bilateral breast
reduction surgery and was found to have lobular carcinoma in situ in the reduction specimen from the left
breast. Her preoperative mammogram was normal, and she has no history of prior breast abnormalities.
The patient should be advised that lobular carcinoma in situ
A. Should be treated like ductal carcinoma in situ.
B. Is an incidental finding in the breast tissue that does not increase the risk of breast cancer.
C. Is a risk factor for the future development of breast cancer only in the breast in which it was found.
D. Is a risk factor for the future development of breast cancer in both breasts.
E. Is a precursor to invasive lobular breast cancer and should be treated by mastectomy.
ANSWER: D
82. All the following are true about testicular torsion except
A. It occurs most often in adolescents.
B. It occurs usually outside the tunica vaginalis.
C. The pain is not always scrotal.
D. The torsion must be relieved within 6 hours from the onset of pain.
E. After detorsion, bilateral orchidopexy is indicated.
ANSWER: B
83. All the following are true regarding infantile hypertrophic pyloric stenosis are true except
A. Associated with bilious vomiting.
B. Occur mostly between 2 and 8 weeks of age.
C. More in males.
D. After a feed, a visible gastric peristalsis from left to right can be seen.
E. The diseased infant develops hypochloremic hypokalemic alkalosis.
ANSWER: A
85. All the following are compatible with Milan criteria for the indications of liver transplantation for patients with
hepatocellular carcinoma except
A. Single tumor up to 5cm in longest dimension.
B. Up to 3 tumors the largest is up to 3cm in longest dimension.
C. Absence of gross vascular invasion.
D. Absence of extrahepatic spread.
E. Absence of cirrhosis.
ANSWER: E
86. All the following are true about gallbladder hydrops except
A. It is caused by an impacted stone in the cystic duct.
B. The gallbladder is distended and may be palpable.
C. The gallbladder is full with mucous without bile.
D. The patient usually has right upper abdominal tenderness.
E. Cholecystectomy is indicated.
ANSWER: D
87. The use of noradrenaline gives the best response in shock due to
A. Sever pancreatitis.
B. Anaphylaxis.
C. Sever GI bleeding.
D. Cardiac tamponade.
E. Acute heart failure.
ANSWER: B
88. Which of the following is not part from the intrinsic-common pathway of coagulation
A. Factor II.
B. Factor V.
C. Factor VII.
D. Factor VIII.
E. Factor XI.
ANSWER: C
89. In the genetic model for colorectal tumorigenesis, which of the following genetic hits occur latest?
A. APC mutation.
B. DNA methylation.
C. Loss of p53.
D. K-ras mutation.
E. Loss of DCC.
ANSWER: C
91. All the following are true regarding pressure ulcers except
A. Ischemia results if the pressure on the tissue exceeds the pressure of the microcirculation (30 mmHg).
B. Ischial tuberosities are the commonest areas for pressure ulcers.
C. Skin is more susceptible than muscle to pressure related ischemic injury.
D. Relief of pressure is a corner-stone in management.
E. Antibiotic treatment is not indicated in many cases.
ANSWER: C
92. The breast receives blood supply from all the following arteries except
A. Perforating branches from the internal mammary artery.
B. Branches from the anterior intercostal arteries.
C. The highest thoracic artery.
D. Lateral thoracic artery.
E. The brachial artery.
ANSWER: E
93. All the following are true about invasive lobular carcinoma of the breast except
A. It is less common than invasive ductal carcinoma.
B. It is commonly bilateral.
C. It is commonly multifocal.
D. About 30% express estrogen receptors.
E. It has insidious growth pattern.
ANSWER: D
95. All the following increase the risk for thrombo-embolic phenomena except
A. Factor V Leiden.
B. Antithrombin deficiency.
C. Protein C deficiency.
D. Protein S deficiency.
E. Factor XI elevation.
ANSWER: E
96. All the following are true about desmoid tumors of the abdominal wall except
A. They originate from the Musculo-aponeurotic structures of the anterior abdominal wall.
B. They infiltrate locally their surroundings.
C. They rarely metastasize distally.
D. They may associate familial adenomatosis polyposis FAP.
E. Surgical excision with safety margin is indicated if feasible.
ANSWER: C
97. In Lichtenstein repair of inguinal hernia, which of the following techniques is indicated to decrease medial
recurrence of the hernia?
A. The lateral border of the mesh is split above and below the spermatic cord.
B. The medial aspect of the mesh is fixed to the pubic tubercle.
C. The mesh is fixed to the shelving edge of the inguinal ligament.
D. The medial edge of the mesh is sutured to the anterior rectus sheath.
E. The mesh is tailored to fit well around the spermatic cord but not too tight.
ANSWER: D
98. All the following are true about the spleen except
A. Most accessory spleens locate in the splenic hilum and vascular pedicle.
B. Most splenic ligaments are avascular.
C. In most cases the tail of pancreas locates either abutting the spleen or within 1 cm of the hilum.
D. In about 70% of people the splenic artery divides near its hilum into short terminal branches.
E. It receives part of its blood supply from the short gastric vessels.
ANSWER: D
99. All the following are true about acute cholecystitis except
A. The vast majority is caused by gallstones.
B. The initiating event is typically a stone of the cystic duct.
C. Initially acute cholecystitis is an inflammation.
D. Increase prostaglandin synthesis amplifies the inflammatory process.
E. Secondary bacterial infection happens in 75% of cases.
ANSWER: E
5th year:
2. The most important prognostic factor for sarcomas devoid of metastases is:
A. Node status
B. Size of the tumor
C. Tumor grade
D. Gender of the patient
E. Age of the patient
ANSWER: C
3. The most common malignancy following transplantation is:
A. Lung cancer
B. Prostate cancer
C. Breast cancer
D. Hepatocellular cancer
E. Skin cancer
ANSWER: E
4. Parkland formula is used to calculate fluid requirements of burn patients. For a 70-kg adult with a 40%
second degree burn, the total fluid requirements in the first eight hours is:
A. 2.8 liters
B. 4.2 liters
C. 5.6 liters
D. 7.6 liters
E. 12.8 liters
ANSWER: C
7. The least indicated surgical technique among the following nowadays is:
A. Hemithyroidectomy.
B. Subtotal thyroidectomy.
C. Near-total thyroidectomy.
D. Total thyroidectomy.
E. Total thyroidectomy and central neck dissection.
ANSWER: B
8. One of the following structures is not an anatomical relation to the submandibular salivary gland:
A. The Mylohyoid muscle.
B. The facial artery.
C. The inferior alveolar nerve.
D. The lingual nerve.
E. The hypoglossal nerve.
ANSWER: C
9. Regarding primary lung tumor, all of the following statements are correct except:
A. Lifetime cigarette smoking is a major risk factor for bronchial carcinoma.
B. Small-cell lung cancer is less common, metastases early and is less amenable to surgery compared with
non-small-cell cancer.
C. Finger clubbing and hypertrophic pulmonary osteoarthropathy are usually incidental findings and not due to
primary lung cancer.
D. The appropriate treatment strategy is dependent on tumor type, tumor stage, and the general fitness and
lung function of the patient.
E. Late survival has a direct relationship with the tumor stage at the time of treatment.
ANSWER: C
10. All the following are true about hepatic adenomas except
A. More common in females.
B. Associated with contraceptive pills.
C. Risk of rupture and bleeding is very low.
D. It carries a risk of malignant transformation.
E. Surgical resection is indicated.
ANSWER: C
11. Which of the following is one of the Ranson’s prognostic signs of acute gallstone pancreatitis?
A. Age more than 70 years.
B. WBC more than 16000 per cubic mm.
C. Blood glucose more than 200 mg per dL.
D. Serum LDH more than 350 IU per L.
E. Blood urea nitrogen elevation more than 5 mg per dL within the initial 48 hours.
ANSWER: A
15. All of the following measures have been recommended for control of acid secretion in patients with
Zollinger-Ellison syndrome except:
A. Antrectomy.
B. Highly selective vagotomy.
C. Total gastrectomy.
D. Vagotomy and pyloroplasty.
E. Medical therapy with omeprazole.
Answer: A
16. Common sites of GIST in descending order as per incidence are
A. Ileum–duodenum–rectum–stomach
B. Stomach–ileum–duodenum–rectum
C. Rectum–ileum–duodenum–stomach
D. Duodenum– stomach–ileum–rectum
E. Ileum-rectum-stomach-duodenum
Answer: B
17. One specific problem that may arise with persistent vomiting after any of the bariatric operations is
Wernicke’s encephalopathy, which can be treated with parenteral:
A. Vitamin B12
B. Omeprazole
C. Thiamine (vitamin B1)
D. Ascorbic acid (vitamin C)
E. Scopolamine
Answer: C
18. A full-term baby is born with drooling, coughing, and cyanosis after the first feeding, but these resolve
quickly and spontaneously. The next step in management should be:
A. Immediate intubation
B. Placement of orogastric tube
C. Two-view abdominal x-ray
D. Two-view chest x-ray
E. Upper gastrointestinal (UGI) contrast series
ANSWER: B
19. A 2-year-old child presents with an abdominal mass, “raccoon eyes,” and “blueberry muffin” skin lesions.
This most likely represents:
A. Rhabdomyosarcoma
B. Neuroblastoma
C. Wilms tumor
D. Hepatoblastoma
E. Teratoma
ANSWER: B
20. After completion of surgery, an anticholinesterase is administered to reverse the neuromuscular blockade.
Which of the following muscles would be expected to recover first?
A. Diaphragm
B. Adductor pollicis
C. Ocular muscles
D. Pharyngeal
E. Quadriceps femoris.
ANSWER: A
21. Which of the following statements regarding focused abdominal sonography for trauma (FAST) is true?
A. It can reliably evaluate the retroperitoneum.
B. It can quickly detect the presence of pericardial fluid or a pleural effusion.
C. It is useful in detecting a cardiac contusion.
D. It is considered a replacement for computed tomography (CT).
E. It can reliably detect diaphragmatic injuries.
ANSWER: B
22. According to the Joint Commission, what is the leading cause of sentinel events (medical errors)?
A. Training errors
B. Systems errors
C. Communication errors
D. Cognitive errors
E. Medical administration errors
ANSWER: C
23. The mucous membrane lining the upper half of the anal canal is
A. lined with stratified squamous epithelium.
B. Drained by the inferior rectal vein
C. Drained into the superficial inguinal lymph nodes
D. Sensitive to touch and to pain
E. Sensitive to stretch
ANSWER E
26. Compared with sporadic breast cancers, which of the features listed below is least commonly associated
with the breast cancers found in women with a BRCA 1 mutation?
A. High mitotic activity
B. Medullary type
C. Histologically grade 3
D. Predominant lymphocytic infiltrate
E. Strong estrogen receptor positivity
ANSWER: E
27. Regarding successful day case surgery, one of the following is not true;
A. minimal access techniques.
B. Good hemostasis.
C. Avoidance of unnecessary tissue handling or tension.
D. Provided with adequate analgesia.
E. Major procedures should be scheduled late on list.
ANSWER E
32. One of the following is part of Model for End-stage Liver Disease score
A. Serum albumin.
B. Hematocrit.
C. Platelet count.
D. Serum creatinine.
E. Ascites.
Answer: D
33. All the following are true about Transjugular Intrahepatic Portosystemic Shunt (TIPS) except
A. Its main indication is to stop esophageal variceal hemorrhage not responded to medical treatment.
B. It doesn’t affect possible future liver transplant plan.
C. In improves hepatic encephalopathy.
D. It may induce renal failure.
E. Shunt thrombosis rate is high.
Answer: C
34. All the following are common findings in patients with Budd-Chiari syndrome except
A. Occlusion of the three main hepatic veins.
B. Centrilobular necrosis in liver biopsy.
C. Atrophy of Caudate lobe.
D. Decreased liver perfusion via the portal vein.
E. Hepatomegaly.
Answer: D
37. A 45-year-old man with a 20-year history of quiescent ulcerative colitis undergoes surveillance endoscopy
and is found to have a focus of low-grade dysplasia on random colonic biopsy in the transverse colon. The
most appropriate next step in management is:
A. Transverse colectomy.
B. Extended right hemicolectomy.
C. Total proctocolectomy with end ileostomy.
D. Total proctocolectomy with immediate J-pouch construction.
E. Transverse colectomy and left hemicolectomy.
ANSWER D
38. A patient with malignant mass inquires about tumor biology. One of the following is true
A. Most malignant cells generally arise from multiple cell origin, and undergo malignant transformation to form
a malignant clone
B. Malignant cells proliferate slower than normal cells, but the rate of proliferation becomes faster as the
tumor size increases
C. Malignant cells are characterized by reversion to more primitive forms, with cellular pleomorphism and loss
of contact inhibition
D. Tumor doubling time is constant among different types and require 1 year at most to detect from onset of
malignant transformation
E. Anaplasia is the mildest form of malignancy
Answer: C
39. Neoplastic cells have the ability to spread. One of the following is true:
A. Lymph node metastases permeate the sinusoids of the node and later spread throughout the subcapsular
space
B. Carcinoma in situ has no detectable invasion beyond the basement membrane
C. Lymphatic involvement is common with sarcomas
D. The metastatic process of malignancy is highly efficient, as once they shed into the bloodstream, they
produce metastasis almost always
E. Epithelial neoplasm spread only through hematogenous route
Answer: B
40. A 3-year-old boy is referred to the clinic with a scrotal swelling. On examination, the mass does not
transilluminate and it is impossible to palpate normal cord above it. What is the most likely diagnosis?
A. Hydrocele
B. Direct inguinal hernia
C. Indirect inguinal hernia
D. Femoral hernia
E. Lymphatic malformation
ANSWER: C
41. Which of the following will increase the length of a scar the most?
A. 30-degree Z-plasty.
B. 60-degree Z-plasty.
C. W-plasty.
D. M-plasty.
E. 10-degree Z-plasty.
ANSWER: B
42. Twenty-four hours after colon resection, urine output in a 70-year-old man is 10 mL/h. Blood chemistry
analysis reveals sodium, 138 mEq/L; potassium, 6 mEq/L; chloride, 100 mEq/L; bicarbonate, 14 mEq/L.
His metabolic abnormality is characterized by which of the following?
A. Abdominal distension
B. Peaked T waves
C. Narrow QRS complex
D. Cardiac arrest in systole
E. J wave or Osborne wave
ANSWER: B
43. A 10-year-old boy with history of prolonged bleeding after minor injury is scheduled for inguinal hernia
surgery. The bleeding time, PT, and fibrinogen are normal. What would be the most helpful investigation?
A. Fibrinolysis (euglobulin clot lysis time)
B. Platelet count
C. Thrombin time
D. Partial thromboplastin time (PTT)
E. Factor VII assay
ANSWER: D
44. from a hematologic viewpoint, the loss of which immune function performed by the spleen makes patients
susceptible to overwhelming post-splenectomy infection (OPSI)?
A. Loss of IgA.
B. Loss of IgM.
C. Loss of IgG.
D. Loss of IgE.
E. Loss of IgD.
ANSWER B
46. A 23-year-old man presents to the emergency room with a stab wound to his anterior border of the
sternocleidomastoid muscle, 1 cm above the cricoid cartilage with penetration to the platysma. His vitals
and oxygen saturation are normal, with no other physical findings. Which of the following is considered best
management?
A. Discharge the patient home, as everything seems normal
B. Observe for 6 hours, if patient drinks well, with no respiratory distress then discharge
C. Observe without testing for 24 hours. If no further complaints, then discharge
D. Perform a CT-angiogram. If no carotid injury, then can be discharged
E. Formal neck exploration or perform upper endoscopy with esophagogram and angiogram
Answer: E
47. One of the following is not a risk factor for gastric cancer:
A. Pernicious anemia
B. Helicobacter Pylori
C. Partial gastrectomy
D. Blood group O
E. Dried fish
ANSWER: D
48. The most important step in management of Zenker’s diverticulum of the esophagus is:
A. Esophagectomy
B. Excision of the diverticulum
C. Division of the cricopharyngeus muscle
D. Observation
E. Division of the superior laryngeal constrictor muscles
ANSWER: C
49. A 25-year-old male presents with symptoms of obstructed defecation. On examination he is found to have
a small full thickness external rectal prolapse. The most appropriate management for following patient is
A. Resectional rectopexy
B. Laparoscopic ventral mesh rectopexy
C. Stapled transanal resectional rectopexy
D. Anterior resection
E. Banding of prolapse
ANSWER: C
50. All the following regarding abdominal aortic aneurysm are true except
A. It is the most common type of large vessel aneurysm.
B. The vast majority are associated with atheromatous degeneration.
C. The vast majority starts above the origin of renal arteries.
D. Most remain asymptomatic until rupture occurs.
E. Aortoenteric fistula is a rare presentation.
ANSWER: C
51. One of the followings is used in the treatment of Conn`s syndrome due to adrenal hyperplasia:
A. Mitotane.
B. Spironolactone.
C. Metyrapone .
D. Ketoconazole.
E. Prednisolone.
ANSWER: B
53. The most appropriate management of diverticulitis with associated 3-cm pelvic abscess in stable patient is:
A. Hartmann procedure.
B. Sigmoid resection with primary anastomoses.
C. Parenteral antibiotics.
D. Percutaneous drainage.
E. Exploratory laparotomy.
ANSWER: C
54. Healthy 22-year-old pregnant female who develops Clostridium difficile colitis after antibiotic treatment
should receive which of the following regimens?
A. Per oral metronidazole.
B. Per oral vancomycin.
C. IV metronidazole.
D. IV vancomycin.
E. No treatment in pregnant patients.
ANSWER: B
56. All the following are true about liver hydatid cyst except
A. It is more common in the right liver lobe.
B. The sensitivity of ELISA is about 85%.
C. When possible, complete surgical excision of the cyst without rupture is preferred.
D. Small cysts can be managed by albendazole and observation.
E. Water Lilly appearance indicate high activity of the cyst.
ANSWER: E
57. To prevent hyperacute rejection in solid organ transplant, one of the following is true:
A. Ensure HLA matching is present
B. Give FK 506 (Prograf)
C. Ensure ABO blood group compatibility
D. Give Prednisone
E. Give Azathioprine
ANSWER: C
58. In a trial comparing One Anastomosis Gastric Bypass (OAGB) with Roux-en-Y Gastric Bypass (RYGB),
patients with type II diabetes achieved complete remission in 38% in RYGB VS 60% of in OAGB, with P
value of 0.28. With respect to the P value, one of the following is true:
A. OAGB resulted in a statistically significant higher remission rate
B. OAGB resulted in higher remission rate but did not reach statistical significance
C. OAGB resulted in a higher remission rate with less than 5% probability to be due to chance
D. P value in this scenario indicates the results are well-powered
E. P value actually indicates results of RYGB are better
ANSWER: B
60. In thyroid surgery; routine central neck dissection is an essential part in the treatment of:
A. Papillary thyroid carcinoma.
B. Follicular thyroid carcinoma.
C. Medullary thyroid carcinoma.
D. Anaplastic carcinoma.
E. Thyroid Lymphoma.
ANSWER: C
62. In a patient with a zone 3 penetrating neck injury presented to the emergency room with neck hematoma
and external bleeding through the wound controlled by pressure, he is fully conscious and has stable vital
signs. The next step is:
A. Admission to theatre and neck exploration under general anesthesia.
B. Wound exploration in emergency room under local anesthesia.
C. Angiogram.
D. Bronchoscopy and Esophagoscopy.
E. Admission to an intensive care unit, maintain semi setting position and keep pressure dressing for 24 hours.
ANSWER: C
63. The most common cause for recurrent diverticulitis after sigmoid resection is:
A. Pan colonic diverticular disease.
B. Inadequate distal resection margin.
C. Use of a side-to-end stapled technique.
D. Dietary indiscretion.
E. Early enteral feeding.
ANSWER: B
64. All the following are true regarding alcoholic chronic pancreatitis except
A. It doesn’t occur in patients with low daily consumption.
B. The risk of the disease is high among persons who consume 150 cc or more of alcohol daily.
C. The onset of symptoms typically starts after more than 10 years of alcohol abuse.
D. The disease appears in about 10% of alcohol abusers.
E. Genetic factors may play a role in the development of the disease.
ANSWER: A
66. All of the following carry a higher risk of post ERCP pancreatitis except
A. Female gender.
B. Young age.
C. Very high serum bilirubin.
D. Balloon dilatation of the papillary sphincter.
E. Previous ERCP-related pancreatitis.
ANSWER: C
67. All the following are true about ultrasound guided percutaneous drainage of the gall bladder with a pig-tail
catheter except
A. It is advised to avoid passing the catheter through the liver.
B. It is indicated to decompress a purulent gall bladder in a patient not fit for surgery.
C. When the gall bladder inflammation improves, the catheter is removed.
D. If the general condition of the patient improved, the gall bladder can be removed later.
E. The catheter can be used later for dissolution therapy if indicated.
ANSWER: A
68. In a laparoscopic inguinal hernia repair through a trans-abdominal approach in a male patient, the
consultant surgeon discusses with you what forms the triangle of doom. The medial border of the triangle
represents:
A. Cooper’s ligament
B. Vas deferens
C. Gonadal vessels
D. Iliac vessels
E. Iliopubic tract
ANSWER: B
70. A 30-year-old man, suffers a stab wound medial to the left nipple. He had a weak pulse in the ambulance
10 minutes ago, but has no palpable pulse or recordable blood pressure on your examination, while his
pupils are still reactive. The initial surgical approach is:
A. Laparotomy
B. Left postero-lateral thoracotomy
C. Left clavicular incision
D. Left antero-lateral thoracotomy
E. Right antero-lateral thoracotomy
ANSWER: D
71. A 14-year-old boy is diagnosed with a duodenal hematoma after a blunt trauma. He has no other injuries or
findings. Please choose the true answer pertaining to his diagnosis:
A. Patients present commonly one week after injury
B. Patients generally present with peritonitis
C. Patients almost always need exploration of the hematoma since it is in zone I
D. Patients usually present with obstruction symptoms & need total parenteral nutrition till hematoma resolves
E. Patients generally eat well and can be managed well at home
ANSWER: D
72. A 73-year-old man undergoes a laparotomy for mesenteric infarction. An extensive small bowel resection is
undertaken. A planned re-look laparotomy is scheduled to occur in 24 hours' time. What is the most
appropriate closure strategy in this situation?
A. Mass closure of the abdomen obeying Jenkins rule using 2 PDS
B. Application of a Bogota bag
C. Mass closure of the abdomen obeying Jenkins rule using 2/0 PDS
D. Mass closure of the abdomen obeying Jenkins rule using 2 nylon
E. Mass closure of the abdomen obeying Jenkins rule using 2/0 nylon
ANSWER: B
73. A group of surgeons wish to determine whether patients are receiving adequate deep vein thrombosis
prophylaxis following surgery. What is the most appropriate process to determine this?
A. Departmental review
B. Peer review
C. Financial audit
D. Systems based audit
E. Standards based audit
ANSWER: E
74. Absolute contraindications for bariatric surgery include:
A. Cardiomyopathy
B. Pickwickian syndrome
C. Type 1 diabetes mellitus
D. Nonalcoholic steatotic hepatitis
E. Recent coronary arteries stents
ANSWER: E
75. Which of the following statements regarding management of intussusception in adults is true?
A. Barium enema with pneumatic decompression
B. Exploratory laparotomy, manual reduction and resection of the involved segment
C. Exploratory laparotomy and manual reduction
D. Exploratory laparotomy and intestinal bypass
E. Nasogastric tube placement, intravenous fluid, and trial of nonoperative management
ANSWER: B
77. 76 years old female with 12 months history of dysphagia is admitted to the hospital for pneumonia.
Gastrographin swallow shows a tracheoesopageal fistula. Upper endoscopy biopsies showed squamous
cell carcinoma. What is the preferred treatment?
A. Esophagectomy
B. Cervical esophagostomy and percutaneous endoscopic gastrostomy (PEG) tube placement.
C. Esophageal stent placement
D. Radiotherapy
E. Chemoradiotherapy followed by esophagectomy
ANSWER: C
78. Cytokines involved in the initial proinflammatory response include all of the following except:
A. Interleukin-6
B. Interleukin-10
C. Tumor necrosis factor-α
D. Interleukin-1
E. Interleukin-8
ANSWER: B
79. Which of the following is the best parameter for monitoring septic shock?
A. Central venous pressure (CVP)
B. Vasopressor requirement
C. Urine output
D. Serum lactate
E. Mental status changes
ANSWER: D
80. Which of the following clinical conditions is identified by the presence of antibodies in the serum against
HBsAg (anti- HBs) in the absence of hepatitis B core antigen (anti-HBc) and HBsAg? The patient
A. Is susceptible to HBV infection.
B. Is immune because of HBV vaccination.
C. Has an active acute infection with HBV.
D. Has chronic active hepatitis with HBV.
E. Has recovered from an HBV infection with subsequent natural immunity.
ANSWER: B
81. Which of the following is the primary fuel source in prolonged simple starvation?
A. Liver glycogen
B. Muscle protein
C. Muscle glycogen
D. Ketone bodies
E. Dextrose
ANSWER: D
82. All of the following are true regarding head injury EXCEPT:
A. Epidural hematomas are associated with a lucid interval.
B. Subdural hematomas are due to lacerations of bridging veins.
C. Epidural hematomas are usually associated with a skull fracture.
D. Some small subdural hematomas can be managed nonoperatively.
E. Epidural hematomas generally have a worse prognosis than subdural hematomas.
ANSWER: E
83. One month after an antrectomy with Billroth II reconstruction, a patient presents with colicky abdominal
pain, distention, bilious emesis, and failure to pass gas. This most likely represents:
A. Blind loop syndrome
B. Afferent loop syndrome
C. Reflux gastritis
D. Efferent loop syndrome
E. Vitamin deficiency
ANSWER: D
84. A 49-year-old woman is diagnosed with Paget’s disease of the nipple. What is the most appropriate
treatment?
A. Excision of nipple-areola complex
B. Nipple-areola conserving recentralization surgery
C. Radiotherapy alone
D. Six-monthly mammography follow-up
E. Total duct excision
ANSWER: A
85. Which breast’s pathology, presents with dark, multifocal, painless skin nodules and is most commonly
associated with radiation and long-standing lymphoedema?
A. Primary angiosarcoma of the breast
B. Lymphoma of the breast
C. Melanoma of the breast
D. Secondary angiosarcoma of the breast
E. Pseudoangiomatous stromal hyperplasia
ANSWER: D
86. A 48-year-old woman had established renal failure secondary to diabetic nephropathy, for 10 years, until 1
year ago when she underwent a successful renal transplant. She now presents to acute medical services
with painful white lesions on her shins and forearms, and is found to have a serum calcium level of 3.1
mmol/L. the single most likely diagnosis is?
A. MEN-2B syndrome
B. Pleomorphic adenoma
C. Primary hyperparathyroidism
D. Secondary hyperparathyroidism
E. Tertiary hyperparathyroidism
ANSWER: E
87. When assessing a patient in clinic with a suspected melanoma, which one of the following would be most
in keeping with development of a nodular subtype?
A. Development within a long-standing mole
B. Development de novo
C. Development from a Hutchinson freckle
D. Development within a sun-protected site
E. Development from a Spitz nevus
ANSWER: B
88. All the following are true regarding inguinal hernia and hydrocele in children except
A. Inguinal hernias are more common on the right side.
B. Inguinal hernias are almost always indirect.
C. The majority of hydroceles resolve spontaneously.
D. In infants, scrotal transillumination differentiates well between the two conditions.
E. In many cases the diagnosis of indirect hernia in infants depends upon the history.
ANSWER: D
89. All the following are true regarding the treatment of lower limbs arterial stenosis except:
A. A femoral artery puncture is used to access the vessels.
B. Balloon dilatation is commonly used for short segment stenosis.
C. The results are better when treating vessels below the knee.
D. If the dilated segment fails to keep dilated after the balloon in removed, a metallic stent is applied.
E. Surgical operations are reserved for patients with severe symptoms when angioplasty has failed or is not
possible.
ANSWER: C
91. All the following increase the risk of breast cancer except
A. Intraductal papilloma.
B. Florid hyperplasia.
C. A typical lobular hyperplasia.
D. Lobular carcinoma insitu.
E. Ductal carcinoma insitu.
ANSWER: A
92. All the following are true about BRCA1 mutation except
A. BRCA1 is a tumor suppressor gene.
B. It represents at least 80% of hereditary ovarian cancers.
C. Female mutation carriers have higher lifetime risk to develop breast cancer than ovarian cancer.
D. Most of BRCA1 related breast cancers have positive hormone receptors.
E. It is the commonest known cause of hereditary breast cancer.
ANSWER: D
93. A 44-year-old female has left breast microcalcifications of 6cm in maximum dimension. True-cut biopsy
showed ductal carcinoma insitu. Which of the following represent the best therapeutic option?
A. Wide local excision.
B. Unilateral mastectomy.
C. Bilateral mastectomy.
D. Prophylactic hormonal therapy.
E. Observation.
ANSWER: B
94. All the following are true about abdominal aortic aneurysms AAA except
A. Most are infrarenal.
B. Most are fusiform in shape.
C. Most are due to atherosclerosis.
D. Associated peripheral arterial occlusive disease presents in most cases.
E. CT scan is required for definitive evaluation of AAA.
ANSWER: D
96. A 30-year-old female with idiopathic thrombocytopenic purpura scheduled for splenectomy, her platelets
count is 15000 per cubic mm. what is best recommended to prepare her for splenectomy?
A. High dose pulse steroid therapy.
B. IV immunoglobulin.
C. Rituximab.
D. Thrombopoietin.
E. Thrombopoietin receptor agonist.
ANSWER: B